Git MRCGP 2015
Git MRCGP 2015
Git MRCGP 2015
Question 1 of 117
A 28-year-old woman is diagnosed with constipation predominant irritable bowel syndrome. She
occasionally experiences spasms of pain in the left iliac fossa.
Which one of the following is LEAST likely to help her symptoms?
A. Mebeverine
B. Ispaghula
C. Methylcellulose
D. Sterculia
E. Lactulose
Next question
low-dose tricyclic antidepressants (e.g. amitriptyline 5-10 mg) are used in preference to
selective serotonin reuptake inhibitors
breakfast cereal or porridge) and linseeds (up to one tablespoon per day).
Question 2 of 117
Next
You are asked to review a 78-year-old woman with a non-healing leg ulcer by the practice nurse.
You notice she is very thin. What is the most appropriate tool to screen for malnutrition?
A. GPMS
B. MN-10
C. MUST
D. GP-MN
E. MSPC
Next question
Malnutrition
Malnutrition is an important consequence of and contributor to chronic disease. It is clearly a
complex and multifactorial problem that can be difficult to manage but there are a number of key
points to remember for the exam.
NICE define malnutrition as the following:
Around 10% of patients aged over 65 years are malnourished, the vast majority of those living
independently, i.e. not in hospital or care/nursing homes.
Screening for malnutrition if mostly done using MUST (Malnutrition Universal Screen Tool). A link is
provided to a copy of the MUST score algorithm.
Next
Question 3 of 117
A 35-year-old man who is usually fit and well presents to his GP with a 2 month history of
indigestion. His weight is stable and there is no history of dysphagia. Examination of the abdomen is
unremarkable. Of the following options, what is the most suitable initial management?
A. Urea breath testing and non-urgent referral for endoscopy
B. H pylori eradication therapy and full-dose proton pump inhibitor for three
months
C. Full-dose Proton pump inhibitor and immediate referral for endoscopy
D. Three month course of a standard-dose proton pump inhibitor
E. One month course of a full-dose proton pump inhibitor
Next question
This question highlights the NICE guidelines for the management of dyspepsia.
There is no evidence currently to suggest whether a one month course of a PPI or 'test and treat'
strategy should be adopted first line. Many clinicians prefer to test for H pylori first as this cannot be
done within 2 weeks of acid-suppression therapy, as false-negative results may occur
Given the options available, only the answer is in line with current NICE guidelines
Dyspepsia
In 2014 NICE updated their guidelines for the management of dyspepsia. These take into account
the age of the patient (whether younger or older than 55 years) and the presence or absence of
'alarm signs':
*it is unclear from studies whether a trial of a PPI or a 'test and treat' should be used first
1/3
Next
4.
A 57-year-old woman with a history of gallstones presents with progressive right upper quadrant pain,
rigors and jaundice.
Ascending cholangitis
This is a classical presentation of ascending cholangitis.
5.
A 62-year-old presents with upper abdominal pain. She has recently been discharged from hospital where
she underwent an ERCP to investigate cholestatic liver function tests. The pain is severe. On examination
she is apyrexial and has a pulse of 96 / min.
You answered Ascending cholangitis
The correct answer is Acute pancreatitis
Pancreatitis may develop following an ERCP.
6.
A 76-year-old woman presents with abdominal pain, distension and vomiting. She recently had an
episode of acute cholecystitis and is awaiting a cholecystectomy. She feels her symptoms have returned
over the past few days. On examination her abdomen is distended.
You answered Ascending cholangitis
The correct answer is Gallstone ileus
This patient has developed small bowel obstruction secondary to an impacted gallstone.
Next question
The table below gives characteristic exam question features for conditions causing hepatobiliary
disease and related disorders:
Viral hepatitis
Questions may point to risk factors such as foreign travel or intravenous drug use.
Congestive
hepatomegaly
The liver only usually causes pain if stretched. One common way this can occur is as a
consequence of congestive heart failure. In severe cases cirrhosis may occur.
Biliary colic
RUQ pain, intermittent, usually begins abruptly and subsides gradually. Attacks often occur
after eating. Nausea is common.
It is sometimes taught that patients are female, forties, fat and fair although this is obviously a
generalisation.
Acute cholecystitis
Pain similar to biliary colic but more severe and persistent. The pain may radiate to the back or
right shoulder.
The patient may be pyrexial and Murphy's sign positive (arrest of inspiration on palpation of
the RUQ)
Ascending cholangitis
An infection of the bile ducts commonly secondary to gallstones. Classically presents with a
triad of:
Gallstone ileus
This describes small bowel obstruction secondary to an impacted gallstone. It may develop if a
fistula forms between a gangrenous gallbladder and the duodenum.
Abdominal pain, distension and vomiting are seen.
Cholangiocarcinoma
Persistent biliary colic symptoms, associated with anorexia, jaundice and weight loss. A
palpable mass in the right upper quadrant (Courvoisier sign), periumbilical lymphadenopathy
(Sister Mary Joseph nodes) and left supraclavicular adenopathy (Virchow node) may be seen
Acute pancreatitis
Pancreatic cancer
Painless jaundice is the classical presentation of pancreatic cancer. However pain is actually a
relatively common presenting symptom of pancreatic cancer. Anorexia and weight loss are
common
Typical symptoms are malaise, anorexia and weight loss. The associated RUQ pain tends to be
mild and jaundice is uncommon.
Next
Question 7 of 117
Which one of the following features is more common in Crohn's disease than ulcerative colitis?
A. Abdominal mass palpable in the right iliac fossa
B. Tenesmus
C. Bloody diarrhoea
D. Faecal incontinence
E. Abdominal pain in the left lower quadrant
Next question
The two main types of inflammatory bowel disease are Crohn's disease and Ulcerative colitis. They
have many similarities in terms of presenting symptoms, investigation findings and management
options. There are however some key differences which are highlighted in table below:
Features
Extraintestinal
Pathology
Continuous disease
Histology
Endoscopy
Radiology
Barium enema
loss of haustrations
superficial ulceration, 'pseudopolyps'
long standing disease: colon is narrow and
short -'drainpipe colon'
*impaired bile acid rebsorption increases the loss calcium in the bile. Calcium normally binds
oxalate.
Question 8 of 117
Next
A 30-year-old woman is admitted to hospital with abdominal pain and diarrhoea. She has no past
medical history other than depression for which she takes citalopram. She smokes 20 cigarettes/day
and drinks 20 units of alcohol per week. Ileocolonoscopy shows features consistent with Crohn's
disease and she is treated successfully with glucocorticoid therapy. Which one of the following is the
most important intervention to reduce the chance of further episodes?
A. Infliximab
B. Stop drinking
C. Stop smoking
D. Mesalazine
E. Budesonide
Next question
colon but may be seen anywhere from the mouth to anus. NICE published guidelines on the
management of Crohn's disease in 2012.
General points
some studies suggest an increased risk of relapse secondary to NSAIDs and the combined
oral contraceptive pill but the evidence is patchy
Inducing remission
Maintaining remission
as above, stopping smoking is a priority (remember: smoking makes Crohn's worse, but may
help ulcerative colitis)
Surgery
around 80% of patients with Crohn's disease will eventually have surgery
Next
Question 9 of 117
Which one of the following is the most effective screening tool for harmful alcohol drinking and
alcohol dependence?
A. Liver ultrasound
B. CAGE questionnaire
C. FAST questionnaire
D. Combination of MCV and gamma GT blood test
E. AUDIT questionnaire
Next question
FAST
4 item questionnaire
MEN: How often do you have EIGHT or more drinks on one occasion?
WOMEN: How often do you have SIX or more drinks on one occasion?
How often during the last year have you been unable to remember what happened the night before because you
had been drinking?
How often during the last year have you failed to do what was normally expected of you because of drinking?
In the last year has a relative or friend, or a doctor or other health worker been concerned about your drinking or
suggested you cut down?
CAGE
well known but recent research has questioned it's value as a screening test
two or more positive answers is generally considered a 'positive' result
Have you ever felt you should Cut down on your drinking?
Have you ever had a drink in the morning to get rid of a hangover (Eye opener)?
Diagnosis
ICD-10 definition - 3 or more needed
compulsion to drink
Question 10 of 117
Next
A 35-year-old female presents with abdominal pain associated with bloating for the past 6 months,
Which one of the following symptoms is least associated with a diagnosis of irritable bowel
syndrome?
A. Feeling of incomplete stool evacuation
B. Weight loss
C. Back pain
D. Lethargy
E. Nausea
Next question
Weight loss is not a feature of IBS and underlying malignancy or inflammatory bowel disease needs
to be excluded
Irritable bowel syndrome: diagnosis
NICE published clinical guidelines on the diagnosis and management of irritable bowel syndrome
(IBS) in 2008
The diagnosis of IBS should be considered if the patient has had the following for at least 6 months:
A positive diagnosis of IBS should be made if the patient has abdominal pain relieved by defecation
or associated with altered bowel frequency stool form, in addition to 2 of the following 4 symptoms:
Features such as lethargy, nausea, backache and bladder symptoms may also support the
diagnosis
Red flag features should be enquired about:
rectal bleeding
Next
Question 11 of 117
A 54-year-old woman presents with jaundice shortly after being discharged from hospital. Liver
function tests are reported as follows:
Albumin
49 g/l
Bilirubin
89 mol/l
66 iu/l
245 mol/l
paracetamol
sodium valproate, phenytoin
MAOIs
halothane
anti-tuberculosis: isoniazid, rifampicin, pyrazinamide
statins
alcohol
amiodarone
methyldopa
nitrofurantoin
Liver cirrhosis
methotrexate
methyldopa
amiodarone
Question 12 of 117
Next
A 22-year-old man presents with a three week history of diarrhoea. He says his bowels have not
been right for the past few months and he frequently has to run to the toilet. These symptoms had
seemed to be improving up until three weeks ago. For the past week he has also been passing
some blood in the stool and reports the feeling of incomplete evacuation after going. He has lost no
weight and has a good appetite. Examination of his abdomen demonstrates mild tenderness in the
left lower quadrant but no guarding. What is the most likely diagnosis?
A. Diverticulitis
B. Colorectal cancer
C. Crohn's disease
D. Ulcerative colitis
E. Infective diarrhoea
Next question
Ulcerative colitis
Ulcerative colitis (UC) is a form of inflammatory bowel disease. Inflammation always starts at rectum
(hence it is the most common site for UC), never spreads beyond ileocaecal valve and is continuous.
The peak incidence of ulcerative colitis is in people aged 15-25 years and in those aged 55-65
years.
The initial presentation is usually following insidious and intermittent symptoms. Features include:
bloody diarrhoea
urgency
tenesmus
abdominal pain, particularly in the left lower quadrant
extra-intestinal features (see below)
Questions regarding the 'extra-intestinal' features of inflammatory bowel disease are common:
Notes
Related to disease
activity
Unrelated to disease
activity
Pathology
Barium enema
loss of haustrations
Next
Question 13 of 117
A 78-year-old woman develops profuse, offensive watery diarrhoea following a course of coamoxiclav. A diagnosis of Clostridium difficile diarrhoea is made. On examination she is
haemodynamically stable, apyrexial and has no abdominal signs. What is the most appropriate firstline therapy?
A. Oral vancomycin for 7 days
B. Oral metronidazole for 10-14 days
C. Oral metronidazole + vancomycin for 10-14 days
D. Oral metronidazole for 7 days
E. Probiotic yoghurt for 14 days
Next question
Clostridium difficile
Clostridium difficile is a Gram positive rod often encountered in hospital practice. It produces an
diarrhoea
abdominal pain
a raised white blood cell count is characteristic
if severe toxic megacolon may develop
Question 14 of 117
Next
A 42-year-old woman is investigated for lethargy and diarrhoea. Investigations reveal positive antiendomysial antibodies. Each of the following food stuffs should be avoided, except:
A. Beer
B. Rye
C. Maize
D. Bread
E. Pasta
Next question
barley*: beer
rye
oats**
rice
potatoes
corn (maize)
Tissue transglutaminase antibodies may be checked to check compliance with a gluten free diet.
*whisky is made using malted barley. Proteins such as gluten are however removed during the
distillation process making it safe to drink for patients with coeliac disease
**some patients with coeliac disease appear able to tolerate oats
Question 15 of 117
Next
A 31-year-old woman who initially presented with abdominal pain and constipation is diagnosed with
irritable bowel syndrome. Which one of the following bits of dietary advice is it least suitable to give?
A. Avoid missing meals
B. Restrict tea and coffee to 3 cups per day
Insoluble sources of fibre such as bran and wholemeal should be avoided in IBS
low-dose tricyclic antidepressants (e.g. amitriptyline 5-10 mg) are used in preference to
selective serotonin reuptake inhibitors
breakfast cereal or porridge) and linseeds (up to one tablespoon per day).
Next
Question 16 of 117
A 65-year-old man with liver cirrhosis of unknown cause is reviewed in clinic. Which one of the
following factors is most likely to indicate a poor prognosis?
A. Alanine transaminase > 200 u/l
B. Caput medusae
C. Ascites
D. Gynecomastia
E. Splenomegaly
Next question
Score
Bilirubin (mol/l)
<34
34-50
>50
Albumin (g/l)
>35
28-35
<28
Prothrombin time,
prolonged by (s)
<4
4-6
>6
Encephalopathy
none
mild
marked
Ascites
none
mild
marked
<7=A
7-9 = B
>9=C
Question 17 of 117
Next
A 42-year-old dentist presents to his GP complaining of persistent lethargy. Routine bloods show
abnormal liver function tests so a hepatitis screen is sent. The results are shown below:
Anti-HAV IgG negative
HBsAg
negative
Anti-HBs
positive
Anti-HBc
negative
Anti-HCV
positive
B. Hepatitis C infection
C. Previous vaccination to hepatitis B and C
D. Hepatitis C infection with previous hepatitis B vaccination
E. Hepatitis B and C infection
Next question
Given the deranged liver function tests these results most likely indicate previous hepatitis B
vaccination with active hepatitis C infection. However, around 15% of patients exposed to the
hepatitis C virus clear the infection. It would therefore be necessary to perform a HCV PCR to see if
the virus is still present.
There is currently no vaccination for hepatitis C.
Interpreting hepatitis B serology:
surface antigen (HBsAg) is the first marker to appear and causes the production of anti-HBs
Hepatitis C
Hepatitis C is likely to become a significant public health problem in the UK in the next decade. It is
thought around 200,000 people are chronically infected with the virus. At risk groups include
intravenous drug users and patients who received a blood transfusion prior to 1991 (e.g.
haemophiliacs).
Pathophysiology
Transmission
Features
after exposure to the hepatitis C virus less than 20% of patients develop an acute hepatitis
Complications
chronic infection (80-85%) - only 15-20% of patients will clear the virus after an acute
infection and hence the majority will develop chronic hepatitis C
cirrhosis (20-30% of those with chronic disease)
hepatocellular cancer
cryoglobulinaemia
porphyria cutanea tarda (PCT): it is increasingly recognised that PCT may develop in
patients with hepatitis C, especially if there are other factors such as alcohol abuse
ribavirin - side-effects: haemolytic anaemia, cough. Women should not become pregnant
within 6 months of stopping ribavirin as it is teratogenic
interferon alpha - side-effects: flu-like symptoms, depression, fatigue, leukopenia,
thrombocytopenia
Question 18 of 117
Next
Which one of the following drinks contains the nearest to one unit of alcohol:
A. A 125ml glass of red wine (ABV 12%)
B. A half a pint of beer (ABV 5%)
C. A 25ml single measure of spirits (ABV 40%)
D. A pint of low strength beer (ABV 3.4%)
E. A 125ml glass of white wine (ABV 14%)
Next question
The January 2010 AKT feedback report stated 'Candidates performed poorly in several items
related to alcohol. The subject areas included alcohol units, nutrition, treatments for alcohol
dependence and complications of alcohol abuse other than liver disease. The NHS
Confederation has recently highlighted the increasing burden to the NHS of alcohol related
problems and Candidates require a broad knowledge of this topic. '
Alcohol: units
The government currently recommend the following:
men: should drink no more than 21 units of alcohol per week (and no more than 4 units in
One unit of alcohol is equal to 10 ml of alcohol. The 'strength' of an alcoholic drink is determined by
the 'alcohol by volume' (ABV).
Examples of one unit of alcohol:
To calculate the number of units in a drink multiply the number of millilitres by the ABV and divide by
1,000. For example:
half a 175ml 'standard' glass of red wine = 87.5 * 12 / 1000 = 1.05 units
one bottle of wine = 750 * 12 / 1000 = 9 units
one pint of 5% beer or lager = 568 * 5 / 1000 = 2.8 units
Question 19 of 117
Next
A 43-year-old man with type 2 diabetes mellitus presents with lethargy. His current medications
include metformin and gliclazide, although the gliclazide may soon be stopped due to his obesity. A
number of blood tests are ordered which reveal the following:
HbA1c
8.2%
Bilirubin 23 mol/l
ALP
162 u/l
ALT
120 u/l
AST
109 u/l
On discussing these results he states that he does not drink alcohol. What is the most likely cause of
these abnormal results?
A. Metformin-induced steatohepatitis
B. Haemochromatosis
C. Acute hepatitis secondary to gliclazide
D. Cryptogenic cirrhosis
E. Non-alcoholic fatty liver disease
Next question
By far the most likely diagnosis in an obese type 2 diabetic is non-alcoholic fatty liver disease. This
patient will require a liver screen, ultrasound and liver biopsy to confirm the diagnosis.
A normal ferritin makes a diagnosis of haemochromatosis unlikely, although it should always be
considered in patients with both abnormal LFTs and diabetes.
Non-alcoholic fatty liver disease
Non-alcoholic fatty liver disease (NAFLD) is now the most common cause of liver disease in the
developed world. It is largely caused by obesity and describes a spectrum of disease ranging from:
NAFLD is thought to represent the hepatic manifestation of the metabolic syndrome and hence
insulin resistance is thought to be the key mechanism leading to steatosis
Non-alcoholic steatohepatitis (NASH) is a term used to describe liver changes similar to those seen
in alcoholic hepatitis in the absence of a history of alcohol abuse. It is relatively common and though
to affect around 3-4% of the general population. The progression of disease in patients with NASH
may be responsible for a proportion of patients previously labelled as cryptogenic cirrhosis
Associated factors
obesity
hyperlipidaemia
type 2 diabetes mellitus
jejunoileal bypass
sudden weight loss/starvation
Features
usually asymptomatic
hepatomegaly
ALT is typically greater than AST
increased echogenicity on ultrasound
Management
the mainstay of treatment is lifestyle changes (particularly weight loss) and monitoring
there is ongoing research into the role of gastric banding and insulin-sensitising drugs (e.g.
Metformin)
Question 20 of 117
Next
A 31-year-old man with ulcerative colitis presents with a worsening of his symptoms. He is passing
around four loose stools a day which do not contain blood. He has also experienced some urgency
and tenesmus but is otherwise systemically well. What is the most appropriate management?
A. Rectal mesalazine
B. Oral metronidazole
C. Rectal corticosteroids
D. Observe with review in 7 days time
E. Oral loperamide
Next question
Inducing remission
rectal (topical) aminosalicylates or steroids: for distal colitis rectal mesalazine has been
shown to be superior to rectal steroids and oral aminosalicylates
oral aminosalicylates
oral prednisolone is usually used second-line for patients who fail to respond to
aminosalicylates. NICE recommend waiting around 4 weeks before deciding if first-line
treatment has failed
severe colitis should be treated in hospital. Intravenous steroids are usually given first-line
Maintaining remission
Question 21 of 117
Next
A 72-year-old woman is reviewed following a course of oral flucloxacillin for right lower limb cellulitis.
The local protocol suggest oral clindamycin should be used next-line. Which one of the following
side-effects is it most important to warn her about?
A. Heartburn or indigestion
B. Jaundice
C. Sore throat, bruising or lethargy
D. Avoid any food or drink containing alcohol
E. Diarrhoea
Next question
Clostridium difficile
Clostridium difficile is a Gram positive rod often encountered in hospital practice. It produces an
exotoxin which causes intestinal damage leading to a syndrome called pseudomembranous
colitis. Clostridium difficile develops when the normal gut flora are suppressed by broad-spectrum
antibiotics. Clindamycin is historically associated with causing Clostridium difficile but the aetiology
has evolved significantly over the past 10 years. Second and third generation cephalosporins are
now the leading cause of Clostridium difficile.
Features
diarrhoea
abdominal pain
a raised white blood cell count is characteristic
if severe toxic megacolon may develop
Next
Question 22 of 117
Coeliac disease should be excluded in all patients who are diagnosed with:
A. Splenomegaly
B. Pancreatitis
C. Colon cancer
D. Type 2 diabetes mellitus
E. Graves' disease
Next question
Coeliac disease
Coeliac disease is caused by sensitivity to the protein gluten. Repeated exposure leads to villous
atrophy which in turn causes malabsorption. Conditions associated with coeliac disease include
dermatitis herpetiformis (a vesicular, pruritic skin eruption) and autoimmune disorders (type 1
diabetes mellitus and autoimmune hepatitis). It is strongly associated with HLA-DQ2 (95% of
patients) and HLA-B8 (80%) as well as HLA-DR3 and HLA-DR7
In 2009 NICE issued guidelines on the investigation of coeliac disease. They suggest that the
following patients should be screened for coeliac disease:
Conditions
Complications
0/3
anaemia: iron, folate and vitamin B12 deficiency (folate deficiency is more common than
vitamin B12 deficiency in coeliac disease)
hyposplenism
osteoporosis, osteomalacia
lactose intolerance
enteropathy-associated T-cell lymphoma of small intestine
subfertility, unfavourable pregnancy outcomes
rare: oesophageal cancer, other malignancies
Next
Theme: Dysphagia
A. Pharyngeal pouch
B. Achalasia
C. Kaposi's sarcoma
D. Systemic sclerosis
E. Oesophageal cancer
F. Myasthenia gravis
G. Oesophagitis
H. Motor neuron disease
I. Oesophageal candidiasis
J. Plummer-Vinson syndrome
For each one of the following scenarios please select the most likely diagnosis:
23.
An 40-year-old man presents with dysphagia. He reports being reasonably well in himself other than an
occasional cough. The dysphagia occurs with both liquids and solids. Clinical examination is normal.
24.
A 55-year-old woman presents with swallowing difficulties for the past 5 weeks. She has also noticed
some double vision
You answered Pharyngeal pouch
The correct answer is Myasthenia gravis
25.
A 42-year-old haemophiliac who is known to be HIV positive presents with pain on swallowing for the
past week. He has been generally unwell for the past 3 months with diarrhoea and weight loss
You answered Pharyngeal pouch
The correct answer is Oesophageal candidiasis
Unfortunately many haemophiliacs contracted HIV and hepatitis C in the 1980's from blood
transfusions. Immunocompromised patients are prone to oesophageal candidiasis. This patient requires
an urgent endoscopy to confirm the diagnosis.
Next question
Dysphagia
The table below gives characteristic exam question features for conditions causing dysphagia:
Oesophageal
cancer
Dysphagia may be associated with weight loss, anorexia or vomiting during eating
Past history may include Barrett's oesophagus, GORD, excessive smoking or alcohol use
Oesophagitis
Oesophageal
There may be a history of HIV or other risk factors such as steroid inhaler use
candidiasis
Achalasia
Pharyngeal pouch
Systemic sclerosis
Other features of CREST syndrome may be present, namely Calcinosis, Raynaud's phenomenon,
oEsophageal dysmotility, Sclerodactyly, Telangiectasia
As well as oesophageal dysmotility the lower oesophageal sphincter (LES) pressure is decreased.
This contrasts to achalasia where the LES pressure is increased
Myasthenia gravis
Globus hystericus
Question 26 of 117
Primary biliary cirrhosis is most characteristically associated with:
A. Anti-nuclear antibodies
B. Anti-ribonuclear protein antibodies
C. Anti-mitochondrial antibodies
D. Rheumatoid factor
Next
IgM
anti-Mitochondrial antibodies, M2 subtype
Middle aged females
Diagnosis
anti-mitochondrial antibodies (AMA) M2 subtype are present in 98% of patients and are
highly specific
smooth muscle antibodies in 30% of patients
raised serum IgM
Management
pruritus: cholestyramine
Next
Question 27 of 117
You wish to screen a patient for hepatitis B infection. Which one of the following is the most suitable
test to perform?
A. HBcAg
B. HBsAg
C. Hepatitis B viral load
D. anti-HBs
E. HBeAg
Next question
A positive anti-HBs would imply immunity through either previous immunisation or disease. A
positive HBsAg implies either acute or chronic hepatitis B.
Hepatitis B serology
Interpreting hepatitis B serology is a dying art form which still occurs at regular intervals in medical
exams. It is important to remember a few key facts:
surface antigen (HBsAg) is the first marker to appear and causes the production of anti-HBs
HBsAg normally implies acute disease (present for 1-6 months)
if HBsAg is present for > 6 months then this implies chronic disease (i.e. Infective)
Anti-HBs implies immunity (either exposure or immunisation). It is negative in chronic
disease
Anti-HBc implies previous (or current) infection. IgM anti-HBc appears during acute or recent
hepatitis B infection and is present for about 6 months. IgG anti-HBc persists
HbeAg results from breakdown of core antigen from infected liver cells as is therefore a
marker of infectivity
Example results
previous hepatitis B (> 6 months ago), not a carrier: anti-HBc positive, HBsAg negative
previous hepatitis B, now a carrier: anti-HBc positive, HBsAg positive
0/3
Theme: Diarrhoea
A. Gastroenteritis
B. Crohn's disease
C. Ulcerative colitis
D. Colorectal cancer
E. Laxative abuse
F. Constipation causing overflow
G. Lactose intolerance
H. Diverticulitis
I. Irritable bowel syndrome
J. Coeliac disease
For each one of the following scenarios please select the most likely diagnosis:
28.
A 24-year-old smoker presents with intermittent diarrhoea for the past 6 months. She feels bloated,
especially around her periods. Bloods tests are normal.
You answered Gastroenteritis
Next
29.
A 23-year-old student is admitted due to a two-week history of bloody diarrhoea. He is normally fit and
well and has not been abroad recently. His CRP is raised at 56 on admission.
You answered Gastroenteritis
The correct answer is Ulcerative colitis
The duration of his symptoms make a diagnosis of gastroenteritis less likely. The presence of blood in
the diarrhoea points to a diagnosis of ulcerative colitis rather than Crohn's.
30.
A 72-year-old woman presents with a two day history of diarrhoea and pain in the left iliac fossa. Her
temperature is 37.8C. She has a past history of constipation.
You answered Gastroenteritis
The correct answer is Diverticulitis
This is a typical history of diverticulitis
Next question
Diarrhoea
The table below gives characteristic features for conditions causing diarrhoea:
Usually acute
Gastroenteritis
Diverticulitis
Antibiotic therapy
Usually chronic
Irritable bowel
syndrome
Extremely common. The most consistent features are abdominal pain, bloating and change in bowel
habit. Patients may be divided into those with diarrhoea predominant IBS and those with
constipation predominant IBS.
Features such as lethargy, nausea, backache and bladder symptoms may also be present
Ulcerative colitis
Bloody diarrhoea may be seen. Crampy abdominal pain and weight loss are also common. Faecal
urgency and tenesmus may be seen
Crohn's disease
Crampy abdominal pains and diarrhoea. Bloody diarrhoea less common than in ulcerative colitis.
Other features include malabsorption, mouth ulcers perianal disease and intestinal obstruction
Colorectal
cancer
Symptoms depend on the site of the lesion but include diarrhoea, rectal bleeding, anaemia and
constitutional symptoms e.g. Weight loss and anorexia
Coeliac disease
In children may present with failure to thrive, diarrhoea and abdominal distension
In adults lethargy, anaemia, diarrhoea and weight loss are seen. Other autoimmune conditions may
coexist
thyrotoxicosis
laxative abuse
appendicitis
Question 31 of 117
A 23-year-old man develops watery diarrhoea whilst travelling in Egypt.
Which one of the following is the most likely responsible organism?
Next
A. Salmonella
B. Shigella
C. Campylobacter
D. Escherichia coli
E. Bacillus cereus
Next question
Gastroenteritis
Gastroenteritis may either occur whilst at home or whilst travelling abroad (travellers' diarrhoea)
Travellers' diarrhoea may be defined as at least 3 loose to watery stools in 24 hours with or without
one of more of abdominal cramps, fever, nausea, vomiting or blood in the stool. The most common
cause is Escherichia coli
Another pattern of illness is 'acute food poisoning'. This describes the sudden onset of nausea,
vomiting and diarrhoea after the ingestion of a toxin. Acute food poisoning is typically caused
by Staphylococcus aureus, Bacillus cereus orClostridium perfringens.
Stereotypical histories
Infection
Typical presentation
Escherichia coli
Giardiasis
Cholera
Bloody diarrhoea
Vomiting and abdominal pain
Staphylococcusaureus
Severe vomiting
Short incubation period
Campylobacter
A flu-like prodrome is usually followed by crampy abdominal pains, fever and diarrhoea which
may be bloody
Complications include Guillain-Barre syndrome
Bacillus cereus
Amoebiasis
Gradual onset bloody diarrhoea, abdominal pain and tenderness which may last for several
weeks
Incubation period
*vomiting subtype, the diarrhoeal illness has an incubation period of 6-14 hours
Question 32 of 117
Next
A 54-year-old man is investigated for dyspepsia. An endoscopy shows a gastric ulcer and a CLO
test done during the procedure demonstrates H. pylori infection. A course of H. pylori eradication
therapy is given. Six weeks after completing treatment the patients comes for review. Unfortunately
his symptoms have not improved. What is the most appropriate test to confirm H. pylori eradication?
A. Culture of gastric biopsy
B. H. pylori serology
C. Hydrogen breath test
D. Urea breath test
E. Stool culture
Next question
It is important to remember that H. pylori serology remains positive following eradication.
A stool antigen test, not culture, may be an appropriate alternative.
Helicobacter pylori: tests
Urea breath test
Serum antibody
Gastric biopsy
Question 33 of 117
Next
A 26-year-old woman who is known to have type 1 diabetes mellitus presents with a three-month
history of diarrhoea, fatigue and weight loss. She has tried excluding gluten from her diet for the past
4 weeks and feels much better. She requests to be tested so that a diagnosis of coeliac disease is
confirmed. What is the most appropriate next step?
A. Check her HbA1c
B. No need for further investigation as the clinical response is diagnostic
C. Check anti-endomysial antibodies
D. Arrange a jejunal biopsy
E. Ask her to reintroduce gluten for the next 6 weeks before further testing
Next question
Serological tests and jejunal biopsy may be negative if the patient is following a gluten-free diet. The
patient should eat some gluten in more than one meal every day for at least 6 weeks before further
testing.
Coeliac disease: investigation
Coeliac disease is caused by sensitivity to the protein gluten. Repeated exposure leads to villous
atrophy which in turn causes malabsorption. Conditions associated with coeliac disease include
dermatitis herpetiformis (a vesicular, pruritic skin eruption) and autoimmune disorders (type 1
diabetes mellitus and autoimmune hepatitis).
Diagnosis is made by a combination of immunology and jejunal biopsy. Villous atrophy and
immunology normally reverses on a gluten-free diet.
NICE issued guidelines on the investigation of coeliac disease in 2009. If patients are already taking
a gluten-free diet they should be asked, if possible, to reintroduce gluten for at least 6 weeks prior to
testing.
Immunology
Jejunal biopsy
villous atrophy
crypt hyperplasia
increase in intraepithelial lymphocytes
lamina propria infiltration with lymphocytes
Rectal gluten challenge has been described but is not widely used
Question 34 of 117
Next
A 44-year-old man is diagnosed with a duodenal ulcer. CLO testing performed during the
gastroscopy is positive for Helicobacter pylori. What is the most appropriate management to
eradicate Helicobacter pylori?
A. Lansoprazole + clindamycin + metronidazole
B. Lansoprazole + amoxicillin + clindamycin
C. Lansoprazole + amoxicillin + clarithromycin
D. Omeprazole + amoxicillin + clindamycin
E. Omeprazole + penicillin + metronidazole
Next question
H. pylori eradication:
The BNF recommends a regimen containing amoxicillin and clarithromycin as first-line therapy
Helicobacter pylori
Helicobacter pylori is a Gram negative bacteria associated with a variety of gastrointestinal
problems, principally peptic ulcer disease
Associations
The role of H pylori in Gastro-oesophageal reflux disease (GORD) is unclear - there is currently no
role in GORD for the eradication of H pylori
Question 35 of 117
Next
A 27-year-old woman presents for review. She describes herself as having 'IBS' and for the past two
years has suffered intermittent bouts of abdominal pain, bloating and loose stools. For the past two
weeks however her symptoms have been much worse. She is now passing around 3-4 watery, grey,
'frothy' stools per day. Her abdominal bloating and cramps have also worsened and she is suffering
from excessive flatulence. Judging by the fitting of her clothes she also feels that she has lost
weight. Some blood tests are ordered:
Hb
10.9 g/dl
Platelets
199 * 109/l
WBC
7.2 * 109/l
Ferritin
15 ng/ml
2.1 nmol/l
The main clues is this question are the anaemia and low ferritin/folate levels, all characteristic of
coeliac disease. The description of the diarrhoea is also typical although some patients may have
more overtly 'fatty' stools.
Why not irritable bowel syndrome? Common things are common and atypical presentations of
common conditions are seen more than typical presentations of less common conditions. The main
reason is the bloods - a low ferritin and folate would not develop with IBS +/- gastroenteritis. Even if
the woman suffered from menorrhagia this would not explain the low folate although it may account
for the anaemia/low ferritin.
Coeliac disease is more common than Crohn's by a factor of around 100. In exams there are also
usually more clues to point towards a diagnosis of Crohn's (e.g. mouth ulcers etc).
Coeliac disease
Coeliac disease is caused by sensitivity to the protein gluten. Repeated exposure leads to villous
atrophy which in turn causes malabsorption. Conditions associated with coeliac disease include
dermatitis herpetiformis (a vesicular, pruritic skin eruption) and autoimmune disorders (type 1
diabetes mellitus and autoimmune hepatitis). It is strongly associated with HLA-DQ2 (95% of
patients) and HLA-B8 (80%) as well as HLA-DR3 and HLA-DR7
In 2009 NICE issued guidelines on the investigation of coeliac disease. They suggest that the
following patients should be screened for coeliac disease:
Complications
Conditions
anaemia: iron, folate and vitamin B12 deficiency (folate deficiency is more common than
vitamin B12 deficiency in coeliac disease)
hyposplenism
osteoporosis, osteomalacia
lactose intolerance
enteropathy-associated T-cell lymphoma of small intestine
subfertility, unfavourable pregnancy outcomes
rare: oesophageal cancer, other malignancies
Question 36 of 117
Next
A 59-year-old female presents to her GP with a two month history of indigestion. She is otherwise
well, has not had a similar episode before and takes no regular medication. Of note there is no
recent weight loss or vomiting and abdominal examination is unremarkable. What is the most
appropriate initial management?
A. Long-term course of a H2 receptor antagonist
B. Lifestyle advice with follow-up appointment in one month
C. Urgent referral for endoscopy
D. One month course of a full-dose proton pump inhibitor
E. Urea breath testing and treat for H pylori if positive
Next question
This patient meets the criteria for urgent referral for endoscopy as she is older than 55 years, has
recent onset, persistent and unexplained symptoms
Dyspepsia
In 2014 NICE updated their guidelines for the management of dyspepsia. These take into account
the age of the patient (whether younger or older than 55 years) and the presence or absence of
'alarm signs':
*it is unclear from studies whether a trial of a PPI or a 'test and treat' should be used first
Next
Question 37 of 117
A 36-year-old man is reviewed in clinic. He has recently been started on mesalazine 400mg tds for
ulcerative colitis. Which one of the following adverse effects is least likely to be attributable to
mesalazine?
A. Interstitial nephritis
B. Headaches
C. Acute pancreatitis
D. Agranulocytosis
E. Infertility
Next question
Oligospermia is seen in patients taking sulphasalazine due to the sulphapyridine moiety, which is not
present in mesalazine
Aminosalicylate drugs
5-aminosalicyclic acid (5-ASA) is released in the colon and is not absorbed. It acts locally as an antiinflammatory. The mechanism of action is not fully understood but 5-ASA may inhibit prostaglandin
synthesis
Sulphasalazine
Mesalazine
Olsalazine
two molecules of 5-ASA linked by a diazo bond, which is broken by colonic bacteria
Question 38 of 117
Next
bloating, and/or
change in bowel habit
A positive diagnosis of IBS should be made if the patient has abdominal pain relieved by defecation
or associated with altered bowel frequency stool form, in addition to 2 of the following 4 symptoms:
abdominal bloating (more common in women than men), distension, tension or hardness
symptoms made worse by eating
passage of mucus
Features such as lethargy, nausea, backache and bladder symptoms may also support the
diagnosis
Red flag features should be enquired about:
rectal bleeding
unexplained/unintentional weight loss
family history of bowel or ovarian cancer
onset after 60 years of age
Question 39 of 117
Next
A 30-year-old woman presents with a three month history of indigestion. There is no history of
weight less, anorexia, dysphagia, vomiting or change in bowel habit and abdominal examination is
unremarkable. Which one of the following may decrease the accuracy of a 13C-urea breath test?
Urea breath test - no antibiotics in past 4 weeks, no antisecretory drugs (e.g. PPI) in past 2 weeks
Serum antibody
Gastric biopsy
Question 40 of 117
A 54-year-old female presents to her GP with fatigue and xerostomia. Bloods tests reveal the
following:
Hb
13.9 g/dl
WBC
6.1 *109/l
Bilirubin 33 mol/l
ALP
292 u/l
ALT
47 u/l
Next
B. Infectious mononucleosis
C. Primary biliary cirrhosis
D. Autoimmune hepatitis
E. Primary Sjogren's syndrome
Next question
IgM
anti-Mitochondrial antibodies, M2 subtype
Middle aged females
The dry mouth is this patient is due to sicca syndrome, which occurs in 70% of cases of primary
biliary cirrhosis. The raised alkaline phosphatase point towards a diagnosis primary biliary cirrhosis
rather than primary Sjogren's syndrome.
Primary biliary cirrhosis: features
Primary biliary cirrhosis is chronic liver disorder typically seen in middle-aged females (female:male
ratio of 9:1). The aetiology is not fully understood although it is thought to be an autoimmune
condition. Interlobular bile ducts become damaged by a chronic inflammatory process causing
progressive cholestasis, which may eventually progress to cirrhosis. The classic presentation is
itching in a middle-aged woman
Clinical features
early: may be asymptomatic (e.g. raised ALP on routine LFTs) or fatigue, pruritus
cholestatic jaundice
hyperpigmentation, especially over pressure points
xanthelasmas, xanthomata
also: clubbing, hepatosplenomegaly
late: may progress to liver failure
Complications
Question 41 of 117
Next
A 35-year-old woman is noticed to be jaundiced. As part of a liver screen the following results are
obtained:
Anti-HBs
Positive
Anti-HBc
Positive
Anti-HBs = Hepatitis B Surface Antibody; Anti-HBc = Hepatitis B Core Antibody; HBs antigen = Hepatitis
B Surface Antigen
Hepatitis B serology
Interpreting hepatitis B serology is a dying art form which still occurs at regular intervals in medical
exams. It is important to remember a few key facts:
surface antigen (HBsAg) is the first marker to appear and causes the production of anti-HBs
Example results
0/3
Next
J. Acute pancreatitis
For each of the following scenarios please select the most likely diagnosis:
42.
A woman who is know to have gallstones presents with pain in her right upper quadrant. On
examination she is not jaundiced and has a temperature of 37.8C. Palpating under the right costal
margin causes her to catch her breath.
You answered Ascending cholangitis
The correct answer is Acute cholecystitis
This scenario describes Murphy's sign.
43.
A 72-year-old man who is known to have heart failure and type 2 diabetes mellitus presents with a
persistent dull ache in his right upper quadrant. Blood tests show a mild elevation of the alanine
aminotransferase level.
You answered Ascending cholangitis
The correct answer is Congestive hepatomegaly
This patient is likely to have congestive hepatomegaly secondary to heart failure.
44.
A 23-year-old student who has recently returned from a trip to North Africa presents with anorexia,
nausea, mild right upper quadrant pain and lethargy. Blood tests show a marked elevation of his alanine
aminotransferase level.
You answered Ascending cholangitis
The correct answer is Viral hepatitis
Next question
The table below gives characteristic exam question features for conditions causing hepatobiliary
disease and related disorders:
Viral hepatitis
Questions may point to risk factors such as foreign travel or intravenous drug use.
Congestive
hepatomegaly
The liver only usually causes pain if stretched. One common way this can occur is as a
consequence of congestive heart failure. In severe cases cirrhosis may occur.
Biliary colic
RUQ pain, intermittent, usually begins abruptly and subsides gradually. Attacks often occur
after eating. Nausea is common.
It is sometimes taught that patients are female, forties, fat and fair although this is obviously a
generalisation.
Acute cholecystitis
Pain similar to biliary colic but more severe and persistent. The pain may radiate to the back or
right shoulder.
The patient may be pyrexial and Murphy's sign positive (arrest of inspiration on palpation of
the RUQ)
Ascending cholangitis
An infection of the bile ducts commonly secondary to gallstones. Classically presents with a
triad of:
Gallstone ileus
This describes small bowel obstruction secondary to an impacted gallstone. It may develop if a
fistula forms between a gangrenous gallbladder and the duodenum.
Abdominal pain, distension and vomiting are seen.
Cholangiocarcinoma
Persistent biliary colic symptoms, associated with anorexia, jaundice and weight loss. A
palpable mass in the right upper quadrant (Courvoisier sign), periumbilical lymphadenopathy
(Sister Mary Joseph nodes) and left supraclavicular adenopathy (Virchow node) may be seen
Acute pancreatitis
Vomiting is common
Examination may reveal tenderness, ileus and low-grade fever
Periumbilical discolouration (Cullen's sign) and flank discolouration (Grey-Turner's sign) is
described but rare
Pancreatic cancer
Painless jaundice is the classical presentation of pancreatic cancer. However pain is actually a
relatively common presenting symptom of pancreatic cancer. Anorexia and weight loss are
common
Typical symptoms are malaise, anorexia and weight loss. The associated RUQ pain tends to be
mild and jaundice is uncommon.
Question 45 of 117
Next
What is the most appropriate advice to give a man with regards to alcohol intake?
A. No more than 14 units of alcohol per week (and no more than 2 units in any
one day)
B. No more than 14 units of alcohol per week (and no more than 4 units in any
one day)
C. No more than 21 units of alcohol per week (and no more than 4 units in any
one day)
D. No more than 21 units of alcohol per week
E. No more than 21 units of alcohol per week (and no more than 3 units in any
one day)
Next question
The January 2010 AKT feedback report stated 'Candidates performed poorly in several items
related to alcohol. The subject areas included alcohol units, nutrition, treatments for alcohol
dependence and complications of alcohol abuse other than liver disease. The NHS
Confederation has recently highlighted the increasing burden to the NHS of alcohol related
problems and Candidates require a broad knowledge of this topic. '
Alcohol: units
The government currently recommend the following:
men: should drink no more than 21 units of alcohol per week (and no more than 4 units in
any one day)
women: should drink no more than 14 units of alcohol per week (and no more than 3 units in
any one day)
One unit of alcohol is equal to 10 ml of alcohol. The 'strength' of an alcoholic drink is determined by
the 'alcohol by volume' (ABV).
Examples of one unit of alcohol:
To calculate the number of units in a drink multiply the number of millilitres by the ABV and divide by
1,000. For example:
half a 175ml 'standard' glass of red wine = 87.5 * 12 / 1000 = 1.05 units
one bottle of wine = 750 * 12 / 1000 = 9 units
one pint of 5% beer or lager = 568 * 5 / 1000 = 2.8 units
Question 46 of 117
A 45-year-old man with a history of alcohol excess is diagnosed as having grade 3 oesophageal
varices during an outpatient endoscopy. Of the following options, what is the most appropriate
management to prevent variceal bleeding?
A. Propranolol
Next
B. Isosorbide mononitrate
C. Endoscopic sclerotherapy
D. Terlipressin
E. Lansoprazole
Next question
Endoscopic sclerotherapy now has little role in the prophylaxis of variceal haemorrhage.
Oesophageal varices
Acute treatment of variceal haemorrhage
Next
Question 47 of 117
You are doing a new patient consultation for an 81-year-old man who has recently joined the
practice. You notice that he is very thin and wonder if he may be malnourished. According to NICE,
what is the cut-off body mass index (BMI) for diagnosing malnutrition?
A. < 17 kg/m
B. < 17.5 kg/m
C. < 18 kg/m
D. < 18.5 kg/m
E. < 19 kg/m
Next question
Malnutrition
Malnutrition is an important consequence of and contributor to chronic disease. It is clearly a
complex and multifactorial problem that can be difficult to manage but there are a number of key
points to remember for the exam.
NICE define malnutrition as the following:
Around 10% of patients aged over 65 years are malnourished, the vast majority of those living
independently, i.e. not in hospital or care/nursing homes.
Screening for malnutrition if mostly done using MUST (Malnutrition Universal Screen Tool). A link is
provided to a copy of the MUST score algorithm.
a 'food-first' approach with clear instructions (e.g. 'add full-fat cream to mashed potato'),
rather than just prescribing oral nutritional supplements (ONS) such as Ensure
if ONS are used they should be taken between meals, rather than instead of meals
Question 48 of 117
Next
According to recent NICE guidelines, which one of the following may have a role in the management
of irritable bowel syndrome?
A. Reflexology
B. Acupuncture
C. Aloe vera
D. Homeopathy
E. Hypnotherapy
Next question
low-dose tricyclic antidepressants (e.g. amitriptyline 5-10 mg) are used in preference to
selective serotonin reuptake inhibitors
breakfast cereal or porridge) and linseeds (up to one tablespoon per day).
Next
Question 49 of 117
Which one of the following regarding the FAST questionnaire for alcohol misuse is correct?
A. May be stopped after first question depending on the answer
B. The minimum score is 4
C. Consists of 5 questions
D. The maximum score is 20
E. The score for hazardous drinking is 7 or more
Next question
FAST
4 item questionnaire
minimum score = 0, maximum score = 16
the score for hazardous drinking is 3 or more
with relation to the first question 1 drink = 1/2 pint of beer or 1 glass of wine or 1 single spirits
if the answer to the first question is 'never' then the patient is not misusing alcohol
if the response to the first question is 'Weekly' or 'Daily or almost daily' then the patient is a
hazardous, harmful or dependent drinker. Over 50% of people will be classified using just
this one question
MEN: How often do you have EIGHT or more drinks on one occasion?
WOMEN: How often do you have SIX or more drinks on one occasion?
How often during the last year have you been unable to remember what happened the night before because you
had been drinking?
How often during the last year have you failed to do what was normally expected of you because of drinking?
In the last year has a relative or friend, or a doctor or other health worker been concerned about your drinking or
suggested you cut down?
CAGE
well known but recent research has questioned it's value as a screening test
two or more positive answers is generally considered a 'positive' result
Have you ever felt you should Cut down on your drinking?
Have you ever had a drink in the morning to get rid of a hangover (Eye opener)?
Diagnosis
ICD-10 definition - 3 or more needed
compulsion to drink
difficulties controlling alcohol consumption
physiological withdrawal
tolerance to alcohol
neglect of alternative activities to drinking
persistent use of alcohol despite evidence of harm
Next
Question 50 of 117
Which one of the following antibiotics is most likely to cause pseudomembranous colitis?
A. Cefaclor
B. Penicillin V
C. Erythromycin
D. Trimethoprim
E. Doxycycline
Next question
Clostridium difficile
Clostridium difficile is a Gram positive rod often encountered in hospital practice. It produces an
exotoxin which causes intestinal damage leading to a syndrome called pseudomembranous
colitis. Clostridium difficile develops when the normal gut flora are suppressed by broad-spectrum
antibiotics. Clindamycin is historically associated with causing Clostridium difficile but the aetiology
has evolved significantly over the past 10 years. Second and third generation cephalosporins are
now the leading cause of Clostridium difficile.
Features
diarrhoea
abdominal pain
a raised white blood cell count is characteristic
if severe toxic megacolon may develop
Next
Primary sclerosing cholangitis is most associated with:
A. Primary biliary cirrhosis
B. Crohn's disease
C. Hepatitis C infection
D. Ulcerative colitis
E. Coeliac disease
Next question
Associations
ulcerative colitis: 4% of patients with UC have PSC, 80% of patients with PSC have UC
Crohn's (much less common association than UC)
HIV
Features
Investigation
ERCP is the standard diagnostic tool, showing multiple biliary strictures giving a 'beaded'
appearance
ANCA may be positive
there is a limited role for liver biopsy, which may show fibrous, obliterative cholangitis often
described as 'onion skin'
Complications
Question 52 of 117
Next
Which one of the following patients is most likely to require screening for hepatocellular carcinoma?
A. A 45-year-old man with liver cirrhosis secondary to hepatitis C
B. A 33-year-old man with HIV. He is taking antiretroviral therapy
Features
Screening with ultrasound (+/- alpha-fetoprotein) should be considered for high risk groups such as:
Management options
liver transplantation
radiofrequency ablation
transarterial chemoembolisation
sorafenib: a multikinase inhibitor
Next
Question 53 of 117
A 30-year-old man comes for review. He returned from a holiday in Egypt yesterday. For the past
two days he has been passing frequent bloody diarrhoea associated with crampy abdominal pain.
Abdominal examination demonstrates diffuse lower abdominal tenderness but there is no guarding
or rigidity. His temperature is 37.5C. What is the most likely causative organism?
A. Giardiasis
B. Enterotoxigenic Escherichia coli
C. Staphylococcus aureus
D. Salmonella
E. Shigella
Next question
Enterotoxigenic Escherichia coli infections do not usually cause bloody diarrhoea. A differential
diagnosis would be amoebic dysentery, enterohemorrhagicEscherichia coli and
possibly Campylobacter.
Gastroenteritis
Gastroenteritis may either occur whilst at home or whilst travelling abroad (travellers' diarrhoea)
Travellers' diarrhoea may be defined as at least 3 loose to watery stools in 24 hours with or without
one of more of abdominal cramps, fever, nausea, vomiting or blood in the stool. The most common
cause is Escherichia coli
Another pattern of illness is 'acute food poisoning'. This describes the sudden onset of nausea,
vomiting and diarrhoea after the ingestion of a toxin. Acute food poisoning is typically caused
by Staphylococcus aureus, Bacillus cereus orClostridium perfringens.
Stereotypical histories
Infection
Typical presentation
Escherichia coli
Giardiasis
Cholera
Shigella
Bloody diarrhoea
Vomiting and abdominal pain
Staphylococcusaureus
Severe vomiting
Short incubation period
Campylobacter
A flu-like prodrome is usually followed by crampy abdominal pains, fever and diarrhoea which
may be bloody
Complications include Guillain-Barre syndrome
Bacillus cereus
Amoebiasis
Gradual onset bloody diarrhoea, abdominal pain and tenderness which may last for several
weeks
Incubation period
*vomiting subtype, the diarrhoeal illness has an incubation period of 6-14 hours
Question 54 of 117
Next
Hepatitis C
Hepatitis C is likely to become a significant public health problem in the UK in the next decade. It is
thought around 200,000 people are chronically infected with the virus. At risk groups include
intravenous drug users and patients who received a blood transfusion prior to 1991 (e.g.
haemophiliacs).
Pathophysiology
Transmission
Features
after exposure to the hepatitis C virus less than 20% of patients develop an acute hepatitis
Complications
chronic infection (80-85%) - only 15-20% of patients will clear the virus after an acute
infection and hence the majority will develop chronic hepatitis C
cirrhosis (20-30% of those with chronic disease)
hepatocellular cancer
cryoglobulinaemia
porphyria cutanea tarda (PCT): it is increasingly recognised that PCT may develop in
patients with hepatitis C, especially if there are other factors such as alcohol abuse
ribavirin - side-effects: haemolytic anaemia, cough. Women should not become pregnant
Next
Question 55 of 117
A 31-year-old woman presents with symptoms consistent with coeliac disease. Which one of the
following tests should be used first-line when screening patients for coeliac disease?
A. Anti-casein antibodies
B. Tissue transglutaminase antibodies
C. Anti-gliadin antibodies
D. Xylose absorption test
E. Anti-endomyseal antibodies
Next question
Tissue transglutaminase antibodies are recommended as the first-line serological test according to
NICE.
Coeliac disease: investigation
Coeliac disease is caused by sensitivity to the protein gluten. Repeated exposure leads to villous
atrophy which in turn causes malabsorption. Conditions associated with coeliac disease include
dermatitis herpetiformis (a vesicular, pruritic skin eruption) and autoimmune disorders (type 1
diabetes mellitus and autoimmune hepatitis).
Diagnosis is made by a combination of immunology and jejunal biopsy. Villous atrophy and
Jejunal biopsy
villous atrophy
crypt hyperplasia
increase in intraepithelial lymphocytes
lamina propria infiltration with lymphocytes
Rectal gluten challenge has been described but is not widely used
Next
A 22-year-old male blood donor is noted to have the following blood results:
Bilirubin
ALP
ALT
Albumin
41 mol/L
84 U/L
23 U/L
41 g/L
B. Dubin-Johnson syndrome
C. Rotor syndrome
D. Hepatitis C infection
E. Infectious mononucleosis
Next question
An isolated hyperbilirubinaemia in a 22-year-old male is likely to be secondary to Gilbert's syndrome.
The normal dipstix urinalysis excludes Dubin-Johnson and Rotor syndrome as these both produce a
conjugated bilirubinaemia. Viral infections are common triggers for a rise in the bilirubin in patients
with Gilbert's
Gilbert's syndrome
Gilbert's syndrome is an autosomal recessive* condition of defective bilirubin conjugation due to a
deficiency of UDP glucuronyl transferase. The prevalence is approximately 1-2% in the general
population
Features
Question 57 of 117
Next
Your next patient is a 34-year-old man who is known to have an alcohol problem. He has drunk
around 100 units per week for the past five years. He regularly misses meals and smokes 20
cigarettes per day. What vitamin supplementation, if any, should you recommend?
A. Oral vitamin B compound
B. Oral thiamine
C. Oral vitamin B compound + multivitamins
D. No supplementation is advised. Give standard dietary advice
E. Oral thiamine + vitamin D
Next question
Whilst vitamin B compound is widely prescribed it is not recommended in recent guidelines, for
example SIGN.
Alcohol - problem drinking: management
Nutritional support
SIGN recommends alcoholic patients should receive oral thiamine if their 'diet may be
deficient'
Drugs used
Next
Question 58 of 117
A 27-year-old female presents with alternating loose and hard stools associated with abdominal
discomfort and bloating. Which one of the following is it most important to do before making a
positive diagnosis of irritable bowel syndrome?
A. Arrange ultrasound abdomen
B. Flexible sigmoidoscopy
C. Ask about family history of ovarian cancer
D. Use a standardised screening tool for depression
E. Perform thyroid function tests
Next question
bloating, and/or
change in bowel habit
A positive diagnosis of IBS should be made if the patient has abdominal pain relieved by defecation
or associated with altered bowel frequency stool form, in addition to 2 of the following 4 symptoms:
Features such as lethargy, nausea, backache and bladder symptoms may also support the
diagnosis
Red flag features should be enquired about:
rectal bleeding
unexplained/unintentional weight loss
family history of bowel or ovarian cancer
onset after 60 years of age
Question 59 of 117
Next
A 51-year-old woman is investigated for lethargy and pruritus. Her appetite is normal and she has
not lost weight. On examination she is not clinically jaundiced and there is no organomegaly. Bloods
tests are reported as follows:
Hb
12.8 g/dl
6.7 * 109/l
Na+
140 mmol/l
K+
3.9 mmol/l
Urea
6.2 mmol/l
Creatinine 68 mol/l
Bilirubin 30 mol/l
ALP
231 u/l
ALT
38 u/l
GT
367 u/l
Albumin 39 g/l
IgM
anti-Mitochondrial antibodies, M2 subtype
Middle aged females
The demographic (middle-aged female), history (lethargy, pruritus) and liver function tests (rise in
ALP and GT) all point to a diagnosis of primary biliary cirrhosis (PBC). Anti-mitochondrial antibodies
are found in 98% of patients with PBC.
Primary biliary cirrhosis
Primary biliary cirrhosis is a chronic liver disorder typically seen in middle-aged females
(female:male ratio of 9:1). The aetiology is not fully understood although it is thought to be an
autoimmune condition. Interlobular bile ducts become damaged by a chronic inflammatory process
causing progressive cholestasis which may eventually progress to cirrhosis. The classic presentation
is itching in a middle-aged woman
Associations
rheumatoid arthritis
systemic sclerosis
thyroid disease
Diagnosis
anti-mitochondrial antibodies (AMA) M2 subtype are present in 98% of patients and are
highly specific
smooth muscle antibodies in 30% of patients
raised serum IgM
Management
pruritus: cholestyramine
fat-soluble vitamin supplementation
ursodeoxycholic acid
liver transplantation e.g. if bilirubin > 100 (PBC is a major indication) - recurrence in graft can
occur but is not usually a problem
Question 60 of 117
Next
A 27-year-old woman with chronic left iliac fossa pain and alternating bowel habit is diagnosed with
irritable bowel syndrome. Initial treatment is tried with a combination of antispasmodics, laxatives
and anti-motility agents. Unfortunately after 6 months there has been no significant improvement in
her symptoms. According to recent NICE guidelines, what is the most appropriate next step?
A. Low-dose tricyclic antidepressant
B. Cognitive behavioural therapy
C. Refer for sigmoidoscopy
D. Trial of probiotics
low-dose tricyclic antidepressants (e.g. amitriptyline 5-10 mg) are used in preference to
selective serotonin reuptake inhibitors
consider limiting intake of high-fibre food (for example, wholemeal or high-fibre flour and
breads, cereals high in bran, and whole grains such as brown rice)
reduce intake of 'resistant starch' often found in processed foods
limit fresh fruit to 3 portions per day
for diarrhoea, avoid sorbitol
for wind and bloating consider increasing intake of oats (for example, oat-based
breakfast cereal or porridge) and linseeds (up to one tablespoon per day).
Question 61 of 117
Next
Crohn's disease
Crohn's disease is a form of inflammatory bowel disease. It commonly affects the terminal ileum and
colon but may be seen anywhere from the mouth to anus.
Pathology
inflammation occurs in all layers, down to the serosa. This is why patients with Crohn's are
prone to strictures, fistulas and adhesions
Crohn's disease typically presents in late adolescence or early adulthood. Features include:
diarrhoea: the most prominent symptom in adults. Crohn's colitis may cause bloody
diarrhoea
abdominal pain: the most prominent symptom in children
perianal disease: e.g. Skin tags or ulcers
extra-intestinal features are more common in patients with colitis or perianal disease
Questions regarding the 'extra-intestinal' features of inflammatory bowel disease are common:
Notes
Related to disease
activity
Unrelated to disease
activity
Question 62 of 117
Next
A 29-year-old man is reviewed. Four weeks ago he presented with a one month history of bloody
diarrhoea. He was previously fit and well prior to this episode. When initially reviewed he was
passing on average four loose stools a day with some visible blood. He was haemodynamically
stable with no fever and bloods showed the following:
Hb
15.2 g/dl
Inducing remission
Maintaining remission
Question 63 of 117
Which of the following drugs is least likely to cause cholestasis?
A. Gliclazide
B. Amiodarone
C. Chlorpromazine
D. Oral contraceptive pill
Next
E. Co-amoxiclav
Next question
paracetamol
sodium valproate, phenytoin
MAOIs
halothane
anti-tuberculosis: isoniazid, rifampicin, pyrazinamide
statins
alcohol
amiodarone
methyldopa
nitrofurantoin
Liver cirrhosis
methotrexate
methyldopa
amiodarone
Question 64 of 117
Next
Ulcerative colitis
Ulcerative colitis (UC) is a form of inflammatory bowel disease. Inflammation always starts at rectum
(hence it is the most common site for UC), never spreads beyond ileocaecal valve and is continuous.
The peak incidence of ulcerative colitis is in people aged 15-25 years and in those aged 55-65
years.
The initial presentation is usually following insidious and intermittent symptoms. Features include:
bloody diarrhoea
urgency
tenesmus
abdominal pain, particularly in the left lower quadrant
extra-intestinal features (see below)
Questions regarding the 'extra-intestinal' features of inflammatory bowel disease are common:
Notes
Related to disease
activity
Unrelated to disease
activity
Pathology
Barium enema
loss of haustrations
superficial ulceration, 'pseudopolyps'
long standing disease: colon is narrow and short -'drainpipe colon'
Question 65 of 117
Next
A 54-year-old female presents with a 3 month history of dysphagia affecting both food and liquids
from the start, along with symptoms of heartburn. What is the most likely underlying diagnosis?
A. Pharyngeal pouch
B. Gastric adenocarcinoma
C. Benign stricture
D. Oesophageal cancer
E. Achalasia
Next question
Dysphagia affecting both solids and liquids from the start - think achalasia
This is a classic history of achalasia with dysphagia affecting both solids and liquids from the start
Achalasia
Failure of oesophageal peristalsis and of relaxation of lower oesophageal sphincter (LOS) due to
degenerative loss of ganglia from Auerbach's plexus i.e. LOS contracted, oesophagus above dilated.
Achalasia typically presents in middle-age and is equally common in men and women.
Clinical features
Investigations
manometry: excessive LOS tone which doesn't relax on swallowing - considered most
important diagnostic test
barium swallow shows grossly expanded oesophagus, fluid level, 'bird's beak' appearance
CXR: wide mediastinum, fluid level
Treatment
This film demonstrates the classical 'bird's beak' appearance of the lower oesophagus that is seen in achalasia. An
air-fluid level is also seen due to a lack of peristalsis
Mediastinal widening secondary to achalasia. An air-fluid level can sometimes be seen on CXR but it is not visible on
this film
Barium swallow - grossly dilated filled oesophagus with a tight stricture at the gastroesophageal junction resulting in a
'bird's beak' appearance. Tertiary contractions give rise to a corkscrew appearance of the oesophagus
Question 66 of 117
A 54-year-old female is presents to surgery one week following a cholecystectomy with profuse
diarrhoea. What is the most likely diagnosis?
A. Campylobacter
B. E. coli
C. Clostridium difficile
D. Salmonella
Next
E. Staphylococcus aureus
Next question
Clostridium difficile is the most likely cause as the patient would have been given broad-spectrum
antibiotics at the time of the operation
Clostridium difficile
Clostridium difficile is a Gram positive rod often encountered in hospital practice. It produces an
exotoxin which causes intestinal damage leading to a syndrome called pseudomembranous
colitis. Clostridium difficile develops when the normal gut flora are suppressed by broad-spectrum
antibiotics. Clindamycin is historically associated with causing Clostridium difficile but the aetiology
has evolved significantly over the past 10 years. Second and third generation cephalosporins are
now the leading cause of Clostridium difficile.
Features
diarrhoea
abdominal pain
a raised white blood cell count is characteristic
if severe toxic megacolon may develop
Question 67 of 117
Next
A 36-year-old man presents with dyspepsia. No alarm symptoms are present. This is his first
episode and he has no significant medical history of note. A test-and-treat strategy is agreed upon.
What is the most appropriate investigation to test for Helicobacter pylori?
A. Gastric aspiration + culture
B. CLO test (rapid urease test)
C. Stool culture
D. Hydrogen breath test
E. 13C-urea breath test
Next question
The urea breath test is highly sensitive, specific and non-invasive. There is no indication for an
endoscopy. Stool antigen, rather than culture, is an alternative.
Helicobacter pylori: tests
Urea breath test
Serum antibody
Gastric biopsy
Question 68 of 117
Next
Which one of the following causes of gastroenteritis has the longest incubation period?
A. Campylobacter
B. Bacillus cereus
C. Shigella
D. Giardiasis
E. Salmonella
Next question
Gastroenteritis
Gastroenteritis may either occur whilst at home or whilst travelling abroad (travellers' diarrhoea)
Travellers' diarrhoea may be defined as at least 3 loose to watery stools in 24 hours with or without
one of more of abdominal cramps, fever, nausea, vomiting or blood in the stool. The most common
cause is Escherichia coli
Another pattern of illness is 'acute food poisoning'. This describes the sudden onset of nausea,
vomiting and diarrhoea after the ingestion of a toxin. Acute food poisoning is typically caused
by Staphylococcus aureus, Bacillus cereus orClostridium perfringens.
Stereotypical histories
Infection
Typical presentation
Escherichia coli
Giardiasis
Cholera
Shigella
Bloody diarrhoea
Vomiting and abdominal pain
Staphylococcusaureus
Severe vomiting
Short incubation period
Campylobacter
A flu-like prodrome is usually followed by crampy abdominal pains, fever and diarrhoea
which may be bloody
Complications include Guillain-Barre syndrome
Bacillus cereus
Amoebiasis
Gradual onset bloody diarrhoea, abdominal pain and tenderness which may last for several
Infection
Typical presentation
weeks
Incubation period
*vomiting subtype, the diarrhoeal illness has an incubation period of 6-14 hours
Question 69 of 117
Next
A 52-year-old woman is diagnosed with non-alcoholic steatohepatitis following a liver biopsy. What is
the single most important step to help prevent the progression of her disease?
A. Stop smoking
B. Start statin therapy
C. Eat more omega-3 fatty acids
D. Start sulfonylurea therapy
E. Weight loss
Next question
NAFLD is thought to represent the hepatic manifestation of the metabolic syndrome and hence
insulin resistance is thought to be the key mechanism leading to steatosis
Non-alcoholic steatohepatitis (NASH) is a term used to describe liver changes similar to those seen
in alcoholic hepatitis in the absence of a history of alcohol abuse. It is relatively common and though
to affect around 3-4% of the general population. The progression of disease in patients with NASH
may be responsible for a proportion of patients previously labelled as cryptogenic cirrhosis
Associated factors
obesity
hyperlipidaemia
type 2 diabetes mellitus
jejunoileal bypass
sudden weight loss/starvation
Features
usually asymptomatic
hepatomegaly
ALT is typically greater than AST
increased echogenicity on ultrasound
Management
the mainstay of treatment is lifestyle changes (particularly weight loss) and monitoring
there is ongoing research into the role of gastric banding and insulin-sensitising drugs (e.g.
Metformin)
Question 70 of 117
Next
You are discussing alcohol intake with a middle-aged man who has just been discharged from
hospital after an episode of acute pancreatitis. He currently drinks around 2 litres of cider (ABV 5%)
a day. How many units is that a week?
A. 25 units
B. 70 units
C. 10 units
D. 80 units
E. 100 units
Next question
2000 ml x 5% = 10,000
Divide this figure by 1,000 to get the number of units = 10,000/1,000 = 10 units/day
10 units/day x 7 days = 70 units
The January 2010 AKT feedback report stated 'Candidates performed poorly in several items
related to alcohol. The subject areas included alcohol units, nutrition, treatments for alcohol
dependence and complications of alcohol abuse other than liver disease. The NHS
Confederation has recently highlighted the increasing burden to the NHS of alcohol related
problems and candidates require a broad knowledge of this topic. '
Alcohol: units
The government currently recommend the following:
men: should drink no more than 21 units of alcohol per week (and no more than 4 units in
any one day)
women: should drink no more than 14 units of alcohol per week (and no more than 3 units in
any one day)
One unit of alcohol is equal to 10 ml of alcohol. The 'strength' of an alcoholic drink is determined by
the 'alcohol by volume' (ABV).
Examples of one unit of alcohol:
To calculate the number of units in a drink multiply the number of millilitres by the ABV and divide by
1,000. For example:
half a 175ml 'standard' glass of red wine = 87.5 * 12 / 1000 = 1.05 units
one bottle of wine = 750 * 12 / 1000 = 9 units
one pint of 5% beer or lager = 568 * 5 / 1000 = 2.8 units
Question 71 of 117
Next
You review a 34-year-old man who has had ulcerative colitis for the past 20 years. He describes a
one week history of passing three bloody stools per day. Despite this he is eating well and denies
abdominal pain. Abdominal examination is unremarkable. What is this episode most likely to
represent?
A. Severe exacerbation of ulcerative colitis
B. Infective exacerbation of ulcerative colitis
C. Mild exacerbation of ulcerative colitis
D. Moderate exacerbation of ulcerative colitis
E. Colorectal cancer secondary to longstanding ulcerative colitis
Next question
Mild
Moderate
Severe
No systemic disturbance
Normal erythrocyte sedimentation
rate and C-reactive protein values
fever
tachycardia
abdominal tenderness, distension or
reduced bowel sounds
anaemia
hypoalbuminaemia
Question 72 of 117
Next
A nurse undergoes primary immunisation against hepatitis B. Levels of which one of the following
should be checked four months later to ensure an adequate response to immunisation?
A. Anti-HBs
B. Anti-HBc
C. Hepatitis B viral load
D. HbeAg
E. HBsAg
Next question
It is preferable to achieve anti-HBs levels above 100 mIU/ml, although levels of 10 mIU/ml or more
are generally accepted as enough to protect against infection
Hepatitis B serology
Interpreting hepatitis B serology is a dying art form which still occurs at regular intervals in medical
exams. It is important to remember a few key facts:
surface antigen (HBsAg) is the first marker to appear and causes the production of anti-HBs
Example results
Question 73 of 117
Which one of the following extra-intestinal manifestations of inflammatory bowel disease is much
more common in ulcerative colitis than in Crohn's disease?
A. Pauciarticular arthritis
B. Osteoporosis
C. Episcleritis
D. Erythema nodosum
Next
Features
Extraintestinal
Pathology
Continuous disease
Histology
Endoscopy
Radiology
Barium enema
loss of haustrations
superficial ulceration, 'pseudopolyps'
long standing disease: colon is narrow and
short -'drainpipe colon'
*impaired bile acid rebsorption increases the loss calcium in the bile. Calcium normally binds
oxalate.
Next
Question 74 of 117
You are reviewing a 33-year-old man who was diagnosed with coeliac disease two years ago. He
complains of ongoing diarrhoea, feeling tired and bloating. You suspect poor compliance with a
gluten-free diet but he reports being 'very consistent'. Which one of the following is the most
appropriate method to ascertain this
A. Food diary
B. Tissue transglutaminase antibodies
C. Faecal calprotectin measurement
D. Refer for a duodenal biopsy
E. Anti-endomysial antibodies
Next question
A food diary is unlikely to be useful if the patient already reports following a gluten free diet.
It is of course important to consider referring such patients if they are shown to have good
compliance. Complications such as small bowel lymphoma may need to be excluded.
Coeliac disease: management
The management of coeliac disease involves a gluten-free diet. Gluten containing cereals include:
rice
potatoes
corn (maize)
Tissue transglutaminase antibodies may be checked to check compliance with a gluten free diet.
*whisky is made using malted barley. Proteins such as gluten are however removed during the
distillation process making it safe to drink for patients with coeliac disease
**some patients with coeliac disease appear able to tolerate oats
Question 75 of 117
Next
A baby is born to a mother who is known to have chronic hepatitis B. The mothers latest results are
as follows:
HBsAg Positive
HBeAg Positive
What is the most appropriate strategy for reducing the vertical transmission rate?
A. Give the newborn hepatitis B vaccine + hepatitis B immunoglobulin
B. Give the newborn hepatitis B vaccine
Question 76 of 117
Next
Which one of the following statements best describes the prevention and treatment of hepatitis C?
A. No vaccine is available and treatment is only successful in around 10-15% of
patients
B. No vaccine and no treatment is available
C. A vaccine is available and treatment is successful in around 50% of patients
Hepatitis C
Hepatitis C is likely to become a significant public health problem in the UK in the next decade. It is
thought around 200,000 people are chronically infected with the virus. At risk groups include
intravenous drug users and patients who received a blood transfusion prior to 1991 (e.g.
haemophiliacs).
Pathophysiology
Transmission
Features
after exposure to the hepatitis C virus less than 20% of patients develop an acute hepatitis
Complications
chronic infection (80-85%) - only 15-20% of patients will clear the virus after an acute
infection and hence the majority will develop chronic hepatitis C
cirrhosis (20-30% of those with chronic disease)
hepatocellular cancer
cryoglobulinaemia
porphyria cutanea tarda (PCT): it is increasingly recognised that PCT may develop in
patients with hepatitis C, especially if there are other factors such as alcohol abuse
up to 55% of patients successfully clear the virus, with success rates of around 80% for
some strains
the aim of treatment is sustained virological response (SVR), defined as undetectable
ribavirin - side-effects: haemolytic anaemia, cough. Women should not become pregnant
within 6 months of stopping ribavirin as it is teratogenic
interferon alpha - side-effects: flu-like symptoms, depression, fatigue, leukopenia,
thrombocytopenia
Question 77 of 117
Next
A 23-year-old female with a history of diarrhoea and weight loss has a colonoscopy to investigate
her symptoms. A biopsy is taken and reported as follows:
E. Colorectal cancer
Next question
Melanosis coli
Melanosis coli is a disorder of pigmentation of the bowel wall. Histology demonstrates pigment-laden
macrophages
It is associated with laxative abuse, especially anthraquinone compounds such as senna
Question 78 of 117
Next
A 36-year-old former intravenous drug user is to commence treatment for hepatitis C with interferonalpha and ribavirin. Which of the following adverse effects are most likely to occur when patients are
treated with interferon-alpha?
A. Diarrhoea and transient rise in ALT
B. Cough and haemolytic anaemia
C. Flu-like symptoms and transient rise in ALT
D. Haemolytic anaemia and flu-like symptoms
E. Depression and flu-like symptoms
Next question
Interferon
Interferons (IFN) are cytokines released by the body in response to viral infections and neoplasia.
They are classified according to cellular origin and the type of receptor they bind to. IFN-alpha and
IFN-beta bind to type 1 receptors whilst IFN-gamma binds only to type 2 receptors.
IFN-alpha
produced by leucocytes
antiviral action
useful in hepatitis B & C, Kaposi's sarcoma, metastatic renal cell cancer, hairy cell leukaemia
adverse effects include flu-like symptoms and depression
IFN-beta
produced by fibroblasts
antiviral action
reduces the frequency of exacerbations in patients with relapsing-remitting MS
IFN-gamma
Question 79 of 117
Next
A 29-year-old woman develops severe vomiting four hours after having lunch at a local restaurant.
What is the most likely causative organism?
A. Escherichia coli
B. Shigella
C. Campylobacter
D. Salmonella
E. Staphylococcus aureus
Next question
The short incubation period and severe vomiting point to a diagnosis ofStaphylococcus aureus food
poisoning.
Gastroenteritis
Gastroenteritis may either occur whilst at home or whilst travelling abroad (travellers' diarrhoea)
Travellers' diarrhoea may be defined as at least 3 loose to watery stools in 24 hours with or without
one of more of abdominal cramps, fever, nausea, vomiting or blood in the stool. The most common
cause is Escherichia coli
Another pattern of illness is 'acute food poisoning'. This describes the sudden onset of nausea,
vomiting and diarrhoea after the ingestion of a toxin. Acute food poisoning is typically caused
by Staphylococcus aureus, Bacillus cereus orClostridium perfringens.
Stereotypical histories
Infection
Typical presentation
Escherichia coli
Giardiasis
Cholera
Shigella
Bloody diarrhoea
Vomiting and abdominal pain
Staphylococcusaureus
Severe vomiting
Short incubation period
Campylobacter
A flu-like prodrome is usually followed by crampy abdominal pains, fever and diarrhoea which
may be bloody
Amoebiasis
Gradual onset bloody diarrhoea, abdominal pain and tenderness which may last for several
weeks
Incubation period
*vomiting subtype, the diarrhoeal illness has an incubation period of 6-14 hours
0/3
Next
J. Acute pancreatitis
For each of the following scenarios please select the most likely diagnosis:
80.
A 56-year-old woman who is known to have gallstones presents with severe epigastric pain and
vomiting. On examination she is apyrexial and tender in the epigastrium.
You answered Ascending cholangitis
The correct answer is Acute pancreatitis
Gallstones and alcohol are the two most common causes of acute pancreatitis. The site of pain and
absence of fever point away from a diagnosis of acute cholecystitis.
81.
A 63-year-old man presents with weight loss and anorexia. He denies any abdominal pain but says his
stools have become pale and difficult to flush away. On examination he is noticed to have jaundiced
sclera.
You answered Ascending cholangitis
The correct answer is Pancreatic cancer
Whilst painless jaundice is the classical presentation of pancreatic cancer most studies suggest pain is a
relatively common presenting symptom.
82.
An overweight 47-year-old woman presents with recurrent episodes of pain in the right upper quadrant
which is brought on by eating fatty food.
You answered Ascending cholangitis
The correct answer is Biliary colic
Patients with gallstones are stereotypically fat, female and in their forties.
Next question
The table below gives characteristic exam question features for conditions causing hepatobiliary
disease and related disorders:
Viral hepatitis
Questions may point to risk factors such as foreign travel or intravenous drug use.
Congestive
hepatomegaly
The liver only usually causes pain if stretched. One common way this can occur is as a
consequence of congestive heart failure. In severe cases cirrhosis may occur.
Biliary colic
RUQ pain, intermittent, usually begins abruptly and subsides gradually. Attacks often occur
after eating. Nausea is common.
It is sometimes taught that patients are female, forties, fat and fair although this is obviously a
generalisation.
Acute cholecystitis
Pain similar to biliary colic but more severe and persistent. The pain may radiate to the back or
right shoulder.
The patient may be pyrexial and Murphy's sign positive (arrest of inspiration on palpation of
the RUQ)
Ascending cholangitis
An infection of the bile ducts commonly secondary to gallstones. Classically presents with a
triad of:
Gallstone ileus
This describes small bowel obstruction secondary to an impacted gallstone. It may develop if a
fistula forms between a gangrenous gallbladder and the duodenum.
Abdominal pain, distension and vomiting are seen.
Cholangiocarcinoma
Persistent biliary colic symptoms, associated with anorexia, jaundice and weight loss. A
palpable mass in the right upper quadrant (Courvoisier sign), periumbilical lymphadenopathy
(Sister Mary Joseph nodes) and left supraclavicular adenopathy (Virchow node) may be seen
Acute pancreatitis
Pancreatic cancer
Painless jaundice is the classical presentation of pancreatic cancer. However pain is actually a
relatively common presenting symptom of pancreatic cancer. Anorexia and weight loss are
common
Typical symptoms are malaise, anorexia and weight loss. The associated RUQ pain tends to be
mild and jaundice is uncommon.
Next
Question 83 of 117
How many units of alcohol are in a 750ml bottle of red wine with an alcohol by volume of 12%?
A. 6 units
B. 7 units
C. 8 units
D. 9 units
E. 10 units
Next question
The January 2010 AKT feedback report stated 'Candidates performed poorly in several items
related to alcohol. The subject areas included alcohol units, nutrition, treatments for alcohol
dependence and complications of alcohol abuse other than liver disease. The NHS
Confederation has recently highlighted the increasing burden to the NHS of alcohol related
problems and candidates require a broad knowledge of this topic. '
Alcohol: units
The government currently recommend the following:
men: should drink no more than 21 units of alcohol per week (and no more than 4 units in
any one day)
women: should drink no more than 14 units of alcohol per week (and no more than 3 units in
any one day)
One unit of alcohol is equal to 10 ml of alcohol. The 'strength' of an alcoholic drink is determined by
the 'alcohol by volume' (ABV).
Examples of one unit of alcohol:
To calculate the number of units in a drink multiply the number of millilitres by the ABV and divide by
1,000. For example:
half a 175ml 'standard' glass of red wine = 87.5 * 12 / 1000 = 1.05 units
Question 84 of 117
Which one of the following is not associated with oesophageal cancer?
A. Achalasia
B. Smoking
Next
smoking
alcohol
GORD
Barrett's oesophagus
achalasia
Plummer-Vinson syndrome
rare: coeliac disease, scleroderma
Barium swallow - 5cm irregular narrowing of the mid-thoracic oesophagus with proximal shouldering
Question 85 of 117
Next
A patient presents with gastrointestinal symptoms. Which one of the following features in the history
would be least consistent with making a diagnosis of irritable bowel syndrome?
A. Urgency to open bowels
B. Symptoms made worse by eating
C. 62-year-old female
D. Passage of mucous with stool
E. Bladder symptoms
Next question
Onset after 60 years of age is considered a red flag in the new NICE guidelines.
Irritable bowel syndrome: diagnosis
NICE published clinical guidelines on the diagnosis and management of irritable bowel syndrome
(IBS) in 2008
The diagnosis of IBS should be considered if the patient has had the following for at least 6 months:
A positive diagnosis of IBS should be made if the patient has abdominal pain relieved by defecation
or associated with altered bowel frequency stool form, in addition to 2 of the following 4 symptoms:
abdominal bloating (more common in women than men), distension, tension or hardness
symptoms made worse by eating
passage of mucus
Features such as lethargy, nausea, backache and bladder symptoms may also support the
diagnosis
Red flag features should be enquired about:
rectal bleeding
unexplained/unintentional weight loss
family history of bowel or ovarian cancer
onset after 60 years of age
ESR/CRP
Question 86 of 117
Next
A 38-year-old woman is noticed to be jaundiced. As part of a liver screen the following results are
obtained:
Anti-HBs
Negative
Anti-HBc
Positive
HBs antigen
Positive
Anti-HBs = Hepatitis B Surface Antibody; Anti-HBc = Hepatitis B Core Antibody; HBs antigen = Hepatitis
B Surface Antigen
surface antigen (HBsAg) is the first marker to appear and causes the production of anti-HBs
Example results
previous hepatitis B (> 6 months ago), not a carrier: anti-HBc positive, HBsAg negative
previous hepatitis B, now a carrier: anti-HBc positive, HBsAg positive
0/3
Theme: Diarrhoea
A. Gastroenteritis
B. Crohn's disease
C. Ulcerative colitis
D. Colorectal cancer
E. Laxative abuse
F. Constipation causing overflow
G. Lactose intolerance
H. Diverticulitis
I. Irritable bowel syndrome
J. Coeliac disease
For each one of the following scenarios please select the most likely diagnosis:
Next
87.
A 41-year-old man with cerebral palsy is admitted with abdominal pain and diarrhoea. His carers report
him passing 5-6 watery stools per day for the past four days. On examination he has a mass in the left
side of the abdomen.
You answered Gastroenteritis
The correct answer is Constipation causing overflow
88.
A 37-year-old woman presents with a three week history of diarrhoea and crampy abdominal pains. On
examination she is noted to have a number of perianal skin tags.
You answered Gastroenteritis
The correct answer is Crohn's disease
89.
A 4-year-old boy is investigated for chronic diarrhoea, abdominal bloating and failure to thrive.
You answered Gastroenteritis
The correct answer is Coeliac disease
These are typical presenting features of coeliac disease in children.
Next question
Diarrhoea
The table below gives characteristic features for conditions causing diarrhoea:
Usually acute
Gastroenteritis
Diverticulitis
Antibiotic therapy
Usually chronic
Irritable bowel
syndrome
Extremely common. The most consistent features are abdominal pain, bloating and change in bowel
habit. Patients may be divided into those with diarrhoea predominant IBS and those with
constipation predominant IBS.
Features such as lethargy, nausea, backache and bladder symptoms may also be present
Ulcerative colitis
Bloody diarrhoea may be seen. Crampy abdominal pain and weight loss are also common. Faecal
urgency and tenesmus may be seen
Crohn's disease
Crampy abdominal pains and diarrhoea. Bloody diarrhoea less common than in ulcerative colitis.
Other features include malabsorption, mouth ulcers perianal disease and intestinal obstruction
Colorectal
cancer
Symptoms depend on the site of the lesion but include diarrhoea, rectal bleeding, anaemia and
constitutional symptoms e.g. Weight loss and anorexia
Coeliac disease
In children may present with failure to thrive, diarrhoea and abdominal distension
In adults lethargy, anaemia, diarrhoea and weight loss are seen. Other autoimmune conditions may
coexist
thyrotoxicosis
laxative abuse
appendicitis
Question 90 of 117
Next
Malnutrition
Malnutrition is an important consequence of and contributor to chronic disease. It is clearly a
complex and multifactorial problem that can be difficult to manage but there are a number of key
points to remember for the exam.
NICE define malnutrition as the following:
Around 10% of patients aged over 65 years are malnourished, the vast majority of those living
independently, i.e. not in hospital or care/nursing homes.
Screening for malnutrition if mostly done using MUST (Malnutrition Universal Screen Tool). A link is
provided to a copy of the MUST score algorithm.
Next
Question 91 of 117
Which one of the following patients would it be most suitable to offer a screening test for coeliac
disease to?
A. A patient who is 'tired all the time'
B. A patient with rheumatoid arthritis
C. A patient who has a family history of inflammatory bowel disease
D. A patient with type 2 diabetes mellitus
E. A patient who develops erythema nodosum
Next question
Coeliac disease
Coeliac disease is caused by sensitivity to the protein gluten. Repeated exposure leads to villous
atrophy which in turn causes malabsorption. Conditions associated with coeliac disease include
dermatitis herpetiformis (a vesicular, pruritic skin eruption) and autoimmune disorders (type 1
diabetes mellitus and autoimmune hepatitis). It is strongly associated with HLA-DQ2 (95% of
patients) and HLA-B8 (80%) as well as HLA-DR3 and HLA-DR7
In 2009 NICE issued guidelines on the investigation of coeliac disease. They suggest that the
following patients should be screened for coeliac disease:
Conditions
Complications
anaemia: iron, folate and vitamin B12 deficiency (folate deficiency is more common than
lactose intolerance
enteropathy-associated T-cell lymphoma of small intestine
subfertility, unfavourable pregnancy outcomes
rare: oesophageal cancer, other malignancies
Question 92 of 117
Next
Gallstones
Ethanol
Trauma
Steroids
Mumps (other viruses include Coxsackie B)
Autoimmune (e.g. polyarteritis nodosa), Ascaris infection
Scorpion venom
Hypertriglyceridaemia, Hyperchylomicronaemia, Hypercalcaemia,Hypothermia
ERCP
Drugs (azathioprine, mesalazine*, didanosine, bendroflumethiazide, furosemide,
pentamidine, steroids, sodium valproate)
Next
Question 93 of 117
Which one of the following regarding the AUDIT questionnaire for alcohol misuse is correct?
A. A score of 12 in a man is likely to indicate a low risk of alcohol misuse
B. Consists of 10 items
C. Asks about thoughts of self harm
D. Maximum score is 30
E. Is less sensitive than the CAGE questionnaire
Next question
FAST
4 item questionnaire
minimum score = 0, maximum score = 16
the score for hazardous drinking is 3 or more
with relation to the first question 1 drink = 1/2 pint of beer or 1 glass of wine or 1 single spirits
if the answer to the first question is 'never' then the patient is not misusing alcohol
if the response to the first question is 'Weekly' or 'Daily or almost daily' then the patient is a
hazardous, harmful or dependent drinker. Over 50% of people will be classified using just
this one question
MEN: How often do you have EIGHT or more drinks on one occasion?
WOMEN: How often do you have SIX or more drinks on one occasion?
How often during the last year have you been unable to remember what happened the night before because you
had been drinking?
How often during the last year have you failed to do what was normally expected of you because of drinking?
In the last year has a relative or friend, or a doctor or other health worker been concerned about your drinking or
suggested you cut down?
CAGE
well known but recent research has questioned it's value as a screening test
two or more positive answers is generally considered a 'positive' result
Have you ever felt you should Cut down on your drinking?
Have you ever had a drink in the morning to get rid of a hangover (Eye opener)?
Diagnosis
ICD-10 definition - 3 or more needed
compulsion to drink
difficulties controlling alcohol consumption
physiological withdrawal
tolerance to alcohol
neglect of alternative activities to drinking
persistent use of alcohol despite evidence of harm
Question 94 of 117
Next
A 62-year-old woman with a history of scleroderma is reviewed. For the past few months she has
suffered with recurrent bouts of diarrhoea. During these bouts her stools are pale, bulky and
offensive. She drinks 14 units of alcohol/week. Bloods show the following:
Hb
10.8 g/dl
Platelets
231 * 109/l
WBC
5.4 * 109/l
Ferritin
14 ng/ml
2.2 nmol/l
Na+
142 mmol/l
K+
3.4 mmol/l
Urea
4.5 mmol/l
Creatinine 77 mol/l
Bilirubin 21 mol/l
ALP
88 u/l
ALT
21 u/l
GT
55 u/l
Albumin 36 g/l
bloods show evidence of impaired absorption of some vitamins (B12, folate), nutrients (iron) and
protein (low albumin).
Malabsorption
Malabsorption is characterised by diarrhoea, steatorrhoea and weight loss. Causes may be broadly
divided into intestinal (e.g. villous atrophy), pancreatic (deficiency of pancreatic enzyme production
or secretion) and biliary (deficiency of bile-salts needed for emulsification of fats)
Intestinal causes of malabsorption
coeliac disease
Crohn's disease
tropical sprue
Whipple's disease
Giardiasis
brush border enzyme deficiencies (e.g. lactase insufficiency)
chronic pancreatitis
cystic fibrosis
pancreatic cancer
biliary obstruction
primary biliary cirrhosis
Other causes
Next
Question 95 of 117
A 29-year-old man presents with a 12 day history of watery diarrhoea that developed one week after
returning from India. He had travelled around northern India for two months. On examination he is
apyrexial and his abdomen is soft and non-tender. What is the most likely causative organism?
A. Amoebiasis
B. Giardiasis
C. Campylobacter
D. Shigella
E. Salmonella
Next question
The incubation period and prolonged, non-bloody diarrhoea point towards giardiasis
Gastroenteritis
Gastroenteritis may either occur whilst at home or whilst travelling abroad (travellers' diarrhoea)
Travellers' diarrhoea may be defined as at least 3 loose to watery stools in 24 hours with or without
one of more of abdominal cramps, fever, nausea, vomiting or blood in the stool. The most common
cause is Escherichia coli
Another pattern of illness is 'acute food poisoning'. This describes the sudden onset of nausea,
vomiting and diarrhoea after the ingestion of a toxin. Acute food poisoning is typically caused
by Staphylococcus aureus, Bacillus cereus orClostridium perfringens.
Stereotypical histories
Infection
Typical presentation
Escherichia coli
Giardiasis
Cholera
Shigella
Bloody diarrhoea
Vomiting and abdominal pain
Staphylococcusaureus
Severe vomiting
Short incubation period
Campylobacter
A flu-like prodrome is usually followed by crampy abdominal pains, fever and diarrhoea which
may be bloody
Complications include Guillain-Barre syndrome
Bacillus cereus
Amoebiasis
Gradual onset bloody diarrhoea, abdominal pain and tenderness which may last for several
weeks
Incubation period
*vomiting subtype, the diarrhoeal illness has an incubation period of 6-14 hours
Question 96 of 117
Next
A patient who was an intravenous drug user in the 1990s asks for a hepatitis C test. What is the
most appropriate action?
A. Refer him for pre-test counselling to discuss the pros and cons of testing
B. Advise him that no accurate test is currently available but that he should
undertake normal precautions
C. Arrange an anti-HCV antibody test
D. Arrange a HCV RNA test
E. Refer him to gastroenterology for a liver biopsy
Next question
HCV RNA tests are normally only ordered following a positive antibody test.
Hepatitis C
Hepatitis C is likely to become a significant public health problem in the UK in the next decade. It is
thought around 200,000 people are chronically infected with the virus. At risk groups include
intravenous drug users and patients who received a blood transfusion prior to 1991 (e.g.
haemophiliacs).
Pathophysiology
Transmission
Features
after exposure to the hepatitis C virus less than 20% of patients develop an acute hepatitis
Complications
chronic infection (80-85%) - only 15-20% of patients will clear the virus after an acute
infection and hence the majority will develop chronic hepatitis C
cirrhosis (20-30% of those with chronic disease)
hepatocellular cancer
cryoglobulinaemia
porphyria cutanea tarda (PCT): it is increasingly recognised that PCT may develop in
patients with hepatitis C, especially if there are other factors such as alcohol abuse
ribavirin - side-effects: haemolytic anaemia, cough. Women should not become pregnant
within 6 months of stopping ribavirin as it is teratogenic
interferon alpha - side-effects: flu-like symptoms, depression, fatigue, leukopenia,
thrombocytopenia
Question 97 of 117
Next
Cirrhosis: if early disease, later liver decreases in size. Associated with a non-tender, firm
liver
Malignancy: metastatic spread or primary hepatoma. Associated with a hard, irregular. liver
edge
Right heart failure: firm, smooth, tender liver edge. May be pulsatile
Other causes
viral hepatitis
glandular fever
malaria
abscess: pyogenic, amoebic
hydatid disease
haematological malignancies
haemochromatosis
primary biliary cirrhosis
sarcoidosis, amyloidosis
Question 98 of 117
Next
A man presents with severe vomiting. He reports not being able to keep fluids down for the past 12
hours. You suspect a diagnosis of gastroenteritis and on discussing possible causes he mentions
reheating curry with rice the night before. What is the most likely causative organism?
A. Escherichia coli
B. Campylobacter
C. Salmonella
D. Shigella
E. Bacillus cereus
Next question
Bacillus cereus infection most commonly results from reheated rice.
Gastroenteritis
Gastroenteritis may either occur whilst at home or whilst travelling abroad (travellers' diarrhoea)
Travellers' diarrhoea may be defined as at least 3 loose to watery stools in 24 hours with or without
one of more of abdominal cramps, fever, nausea, vomiting or blood in the stool. The most common
cause is Escherichia coli
Another pattern of illness is 'acute food poisoning'. This describes the sudden onset of nausea,
vomiting and diarrhoea after the ingestion of a toxin. Acute food poisoning is typically caused
by Staphylococcus aureus, Bacillus cereus orClostridium perfringens.
Stereotypical histories
Infection
Typical presentation
Escherichia coli
Giardiasis
Cholera
Shigella
Bloody diarrhoea
Vomiting and abdominal pain
Staphylococcusaureus
Severe vomiting
Short incubation period
Campylobacter
A flu-like prodrome is usually followed by crampy abdominal pains, fever and diarrhoea which
may be bloody
Complications include Guillain-Barre syndrome
Bacillus cereus
Amoebiasis
Gradual onset bloody diarrhoea, abdominal pain and tenderness which may last for several
weeks
Incubation period
*vomiting subtype, the diarrhoeal illness has an incubation period of 6-14 hours
Question 99 of 117
Which one of the following causes of diarrhoea has the shortest incubation period?
A. Salmonella
Next
B. Shigella
C. Campylobacter
D. Escherichia coli
E. Bacillus cereus
Next question
Gastroenteritis
Gastroenteritis may either occur whilst at home or whilst travelling abroad (travellers' diarrhoea)
Travellers' diarrhoea may be defined as at least 3 loose to watery stools in 24 hours with or without
one of more of abdominal cramps, fever, nausea, vomiting or blood in the stool. The most common
cause is Escherichia coli
Another pattern of illness is 'acute food poisoning'. This describes the sudden onset of nausea,
vomiting and diarrhoea after the ingestion of a toxin. Acute food poisoning is typically caused
by Staphylococcus aureus, Bacillus cereus orClostridium perfringens.
Stereotypical histories
Infection
Typical presentation
Escherichia coli
Giardiasis
Cholera
Shigella
Bloody diarrhoea
Vomiting and abdominal pain
Staphylococcusaureus
Severe vomiting
Short incubation period
Campylobacter
A flu-like prodrome is usually followed by crampy abdominal pains, fever and diarrhoea which
may be bloody
Complications include Guillain-Barre syndrome
Bacillus cereus
Amoebiasis
Gradual onset bloody diarrhoea, abdominal pain and tenderness which may last for several
weeks
Incubation period
*vomiting subtype, the diarrhoeal illness has an incubation period of 6-14 hours
Next
E. Flucloxacillin
Next question
paracetamol
sodium valproate, phenytoin
MAOIs
halothane
anti-tuberculosis: isoniazid, rifampicin, pyrazinamide
statins
alcohol
amiodarone
methyldopa
nitrofurantoin
Liver cirrhosis
methotrexate
methyldopa
amiodarone
Next
Each one of the following is a recognised complication of gastro-oesophageal reflux disease, except:
A. Oesophageal carcinoma
B. Barrett's oesophagus
C. Anaemia
D. Achalasia
E. Benign strictures
Next question
if response then offer low dose treatment, possibly on an as-required basis, with a limited
number of repeat prescriptions
if no response then H2RA or prokinetic for one month
Complications
oesophagitis
ulcers
anaemia
benign strictures
Barrett's oesophagus
oesophageal carcinoma
Next
Next question
Crohn's disease
Crohn's disease is a form of inflammatory bowel disease. It commonly affects the terminal ileum and
colon but may be seen anywhere from the mouth to anus.
Pathology
Crohn's disease typically presents in late adolescence or early adulthood. Features include:
diarrhoea: the most prominent symptom in adults. Crohn's colitis may cause bloody
diarrhoea
abdominal pain: the most prominent symptom in children
perianal disease: e.g. Skin tags or ulcers
extra-intestinal features are more common in patients with colitis or perianal disease
Questions regarding the 'extra-intestinal' features of inflammatory bowel disease are common:
Notes
Related to disease
activity
Unrelated to disease
activity
Next
You review a 24-year-old female who has recently been diagnosed with left-sided ulcerative colitis.
Five days ago she was started on high-dose oral mesalazine by the gastroenterologists. There has
yet to be any change in her stool pattern of passing around 3-4 loose motions/day, which have small
amounts of blood in them. She remains systemically well and abdominal examination is
unremarkable. How long should the initial mesalazine treatment be taken for before deciding
whether it has been successful?
A. 7 days
B. 2 weeks
C. 4 weeks
D. 2 months
E. 4 months
Next question
Inducing remission
severe colitis should be treated in hospital. Intravenous steroids are usually given first-line
Maintaining remission
Next
A 68-year-old woman comes back for review. Two weeks ago she presented with pain in her left
knee not responding to paracetamol and was commenced on diclofenac 50mg tds and lansoprazole
30mg od. Shortly afterwards she developed some indigestion which seems to resolve if she skips
the diclofenac dose. She is otherwise asymptomatic and got good pain relief from diclofenac. Clinical
examination is normal. What is the most appropriate action?
A. 13C-urea breath test
B. Stop diclofenac, continue lansoprazole + review in 1 week
C. Switch diclofenac to ibuprofen, continue lansoprazole
D. Refer urgently for endoscopy
E. Admit
Next question
When NICE first published their dyspepsia guidelines there was a policy of referral for any older
patients with new onset dyspepsia. This led to a deluge of referrals and amended guidelines were
published in 2004. This modified approach seems to be supported by a large trial demonstrating a
minimal effect on mortality of routine referral.
This question gives an example of a scenario where NICE would recommend not initially referring a
patient when there is an obvious trigger (NSAID use).
Dyspepsia
In 2014 NICE updated their guidelines for the management of dyspepsia. These take into account
the age of the patient (whether younger or older than 55 years) and the presence or absence of
'alarm signs':
*it is unclear from studies whether a trial of a PPI or a 'test and treat' should be used first
Next
Which one of the following is not associated with non-alcoholic steatohepatitis?
A. Hyperlipidaemia
B. Obesity
C. Sudden weight loss or starvation
D. Jejunoileal bypass
E. Type 1 diabetes mellitus
Next question
NAFLD is thought to represent the hepatic manifestation of the metabolic syndrome and hence
insulin resistance is thought to be the key mechanism leading to steatosis
Non-alcoholic steatohepatitis (NASH) is a term used to describe liver changes similar to those seen
in alcoholic hepatitis in the absence of a history of alcohol abuse. It is relatively common and though
to affect around 3-4% of the general population. The progression of disease in patients with NASH
may be responsible for a proportion of patients previously labelled as cryptogenic cirrhosis
Associated factors
obesity
hyperlipidaemia
type 2 diabetes mellitus
jejunoileal bypass
sudden weight loss/starvation
Features
usually asymptomatic
hepatomegaly
ALT is typically greater than AST
increased echogenicity on ultrasound
Management
the mainstay of treatment is lifestyle changes (particularly weight loss) and monitoring
there is ongoing research into the role of gastric banding and insulin-sensitising drugs (e.g.
Metformin)
Next
What advice should you give a non-pregnant woman who asks how much alcohol can she drink?
A. Should drink no more than 14 units of alcohol per week (and no more than 2
units in any one day)
B. Should drink no more than 14 units of alcohol per week (and no more than 3
units in any one day)
C. Should drink no more than 7-14 units of alcohol per week (and no more than 2
units in any one day)
D. Should drink no more than 21 units of alcohol per week (and no more than 3
units in any one day)
E. Should drink no more than 7 units of alcohol per week (and no more than 2
units in any one day)
Next question
The January 2010 AKT feedback report stated 'Candidates performed poorly in several items
related to alcohol. The subject areas included alcohol units, nutrition, treatments for alcohol
dependence and complications of alcohol abuse other than liver disease. The NHS
Confederation has recently highlighted the increasing burden to the NHS of alcohol related
problems and Candidates require a broad knowledge of this topic. '
Alcohol: units
The government currently recommend the following:
men: should drink no more than 21 units of alcohol per week (and no more than 4 units in
any one day)
women: should drink no more than 14 units of alcohol per week (and no more than 3 units in
any one day)
One unit of alcohol is equal to 10 ml of alcohol. The 'strength' of an alcoholic drink is determined by
the 'alcohol by volume' (ABV).
Examples of one unit of alcohol:
To calculate the number of units in a drink multiply the number of millilitres by the ABV and divide by
1,000. For example:
half a 175ml 'standard' glass of red wine = 87.5 * 12 / 1000 = 1.05 units
one bottle of wine = 750 * 12 / 1000 = 9 units
Next
You follow-up a 42-year-old man who has just completed a two week course
ofHelicobacter pylori therapy for dyspepsia. How soon can you perform a urea breath test to test for
eradication, assuming he has stopped his eradication therapy today and is taking no other
medication?
A. Straight away
B. In 1 weeks time
C. In 2 weeks time
D. In 4 weeks time
E. In 8 weeks time
Next question
Serum antibody
Gastric biopsy
Next
C. 50-60%
D. 5-10%
E. 15-20%
Next question
Hepatitis C
Hepatitis C is likely to become a significant public health problem in the UK in the next decade. It is
thought around 200,000 people are chronically infected with the virus. At risk groups include
intravenous drug users and patients who received a blood transfusion prior to 1991 (e.g.
haemophiliacs).
Pathophysiology
Transmission
Features
after exposure to the hepatitis C virus less than 20% of patients develop an acute hepatitis
Complications
chronic infection (80-85%) - only 15-20% of patients will clear the virus after an acute
infection and hence the majority will develop chronic hepatitis C
cirrhosis (20-30% of those with chronic disease)
hepatocellular cancer
cryoglobulinaemia
porphyria cutanea tarda (PCT): it is increasingly recognised that PCT may develop in
patients with hepatitis C, especially if there are other factors such as alcohol abuse
ribavirin - side-effects: haemolytic anaemia, cough. Women should not become pregnant
Next
A 55-year-old business man presents with a 15 day history of watery, non-bloody diarrhoea
associated with anorexia and abdominal bloating. His symptoms started 4 days after returning from
a trip to Pakistan. On examination he is apyrexial with dry mucous membranes but normal skin
turgor. What is the most likely causative organism?
A. Salmonella
B. Giardia lamblia
C. Shigella
D. Escherichia coli
E. Norovirus
Next question
Although Escherichia coli is the most common cause of travellers' diarrhoea, in this particular case
the length of illness and nature of symptoms (bloating, watery diarrhoea) points to a diagnosis of
Giardiasis.
Giardiasis
Giardiasis is caused by the flagellate protozoan Giardia lamblia. It is spread by the faeco-oral route
Features
often asymptomatic
lethargy, bloating, abdominal pain
non-bloody diarrhoea
chronic diarrhoea, malabsorption and lactose intolerance can occur
stool microscopy for trophozoite and cysts are classically negative, therefore duodenal fluid
aspirates or 'string tests' (fluid absorbed onto swallowed string) are sometimes needed
Next
E. Insulin
Next question
paracetamol
sodium valproate, phenytoin
MAOIs
halothane
anti-tuberculosis: isoniazid, rifampicin, pyrazinamide
statins
alcohol
amiodarone
methyldopa
nitrofurantoin
Liver cirrhosis
methotrexate
methyldopa
amiodarone
Next
Next question
Please see the British Society of Gastroenterology guidelines for more details.
Crohn's disease: management
Crohn's disease is a form of inflammatory bowel disease. It commonly affects the terminal ileum and
colon but may be seen anywhere from the mouth to anus. NICE published guidelines on the
management of Crohn's disease in 2012.
General points
Inducing remission
Maintaining remission
as above, stopping smoking is a priority (remember: smoking makes Crohn's worse, but may
help ulcerative colitis)
azathioprine or mercaptopurine is used first-line to maintain remission
methotrexate is used second-line
5-ASA drugs (e.g. mesalazine) should be considered if a patient has had previous surgery
Surgery
around 80% of patients with Crohn's disease will eventually have surgery
0/3
Theme: Dysphagia
A. Pharyngeal pouch
B. Achalasia
Next
C. Globus hystericus
D. Systemic sclerosis
E. Oesophageal cancer
F. Myasthenia gravis
G. Oesophagitis
H. Motor neuron disease
I. Oesophageal candidiasis
J. Plummer-Vinson syndrome
For each one of the following scenarios please select the most likely diagnosis:
112.
A 54-year-old man presents with a 3 month history of 'heartburn'. He has noticed that swallowing is
painful, particularly when he eats meat or bread. After eating and at night he has an 'unpleasant'
retrosternal sensation. Clinical examination is unremarkable
You answered Pharyngeal pouch
The correct answer is Oesophagitis
113.
A 67-year-old woman presents with a 5 week history of food getting stuck. She is currently treated for
COPD and was recently noted to have a macrocytosis on routine bloods. On a number of occasions
she has vomited during the meal and says she has no taste for food any more.
You answered Pharyngeal pouch
The correct answer is Oesophageal cancer
Smoking (COPD) and alcohol (macrocytosis) are risk factors for oesophageal cancer
114.
Next question
Dysphagia
The table below gives characteristic exam question features for conditions causing dysphagia:
Oesophageal
cancer
Dysphagia may be associated with weight loss, anorexia or vomiting during eating
Past history may include Barrett's oesophagus, GORD, excessive smoking or alcohol use
Oesophagitis
Oesophageal
candidiasis
There may be a history of HIV or other risk factors such as steroid inhaler use
Achalasia
Pharyngeal pouch
Systemic sclerosis
Other features of CREST syndrome may be present, namely Calcinosis, Raynaud's phenomenon,
oEsophageal dysmotility, Sclerodactyly, Telangiectasia
As well as oesophageal dysmotility the lower oesophageal sphincter (LES) pressure is decreased.
This contrasts to achalasia where the LES pressure is increased
Myasthenia gravis
Globus hystericus
Next
A 30-year-old woman presents with abdominal pain that is associated with alternating diarrhoea and
constipation. Which one of the following symptoms is least consistent with a diagnosis of irritable
bowel syndrome?
A. Feeling of incomplete stool evacuation
B. Waking at night due to the pain
C. Abdominal bloating
D. Faecal urgency
E. Passage of mucous with stool
Next question
Pain which wakes a patient at night is not a feature that would be expected in irritable bowel
syndrome.
Irritable bowel syndrome: diagnosis
NICE published clinical guidelines on the diagnosis and management of irritable bowel syndrome
(IBS) in 2008
The diagnosis of IBS should be considered if the patient has had the following for at least 6 months:
A positive diagnosis of IBS should be made if the patient has abdominal pain relieved by defecation
or associated with altered bowel frequency stool form, in addition to 2 of the following 4 symptoms:
Features such as lethargy, nausea, backache and bladder symptoms may also support the
diagnosis
Red flag features should be enquired about:
rectal bleeding
unexplained/unintentional weight loss
family history of bowel or ovarian cancer
onset after 60 years of age
Next
the first few months of treatment. Asymptomatic elevation of the amylase level is seen in up to 10%
of patients
Acute pancreatitis: causes
The vast majority of cases in the UK are caused by gallstones and alcohol
Popular mnemonic is GET SMASHED
Gallstones
Ethanol
Trauma
Steroids
Mumps (other viruses include Coxsackie B)
Autoimmune (e.g. polyarteritis nodosa), Ascaris infection
Scorpion venom
Hypertriglyceridaemia, Hyperchylomicronaemia, Hypercalcaemia,Hypothermia
ERCP
Drugs (azathioprine, mesalazine*, didanosine, bendroflumethiazide, furosemide,
pentamidine, steroids, sodium valproate)
Conditions
Complications
anaemia: iron, folate and vitamin B12 deficiency (folate deficiency is more common than
vitamin B12 deficiency in coeliac disease)
hyposplenism
osteoporosis, osteomalacia
lactose intolerance
enteropathy-associated T-cell lymphoma of small intestine
subfertility, unfavourable pregnancy outcomes
rare: oesophageal cancer, other malignancies